Вы находитесь на странице: 1из 148

Online Coaching for IBPS, SBI Bank Exams

http://www.bankexamportal.com/elearning

WWW.BANKEXAMPORTAL.COM

Download
SBI PO
Previous Year
Exam Papers
e-Book
2010, 2013, 2014

Buy Printed Study Material for IBPS, SBI Bank Exams


http://www.bankexamportal.com/study-kit

Page 1

Online Coaching for IBPS, SBI Bank Exams


http://www.bankexamportal.com/elearning

SBI PO Exam Paper - 2014


Reasoning
(Based on Memory)

Directions (Q. Nos. 1-6) Study the given information and answer the questions. When a word and
number arrangement machine is given an input line or words and numbers, its arranges them
following a particular rule. The following is an illustration of input and rearrangement.

(All the numbers are two-digit numbers)


Input 14 36 methodology 21 of research
59 crucial 98 very 62 is
Step I 98 14 38 methodology 21 of research
59 crucial very is 62
Step II 98 very 14 36 methodology 21 of
59 crucial is research 62
Step III 59 98 very 14 methodology 21 of
crucial is research 62 36
Step IV 59 98 very of 14 21 crucial is
methodology research 62 36
Step V 21 59 98 very of crucial is
methodology research 62 36 14
Step VI 21 59 98 very of is crucial
methodology research 62 36 14
Step VI is the of the above arrangement as the intended arrangements is obtained.
As per the rules followed in the given, steps find out appropriate steps for the given input. Input 65 work in
23 to be 13 89 managed 48 97 load 34 healthy 55 style

1. Which element is exactly between style and be in second last


steps of the given arrangements?
(1) Work
(2) Healthy
(3) 23
(4) Load
(5) Be

2. What is the position of 46 from the right end in the third steps?

Buy Printed Study Material for IBPS, SBI Bank Exams


http://www.bankexamportal.com/study-kit

Page 2

Online Coaching for IBPS, SBI Bank Exams


http://www.bankexamportal.com/elearning
(1) Sixth
(2) Seventh
(3) Fifth
(4) Eight
(5) Ninth

3. Which of the following is fifth step of the arrangement based on


the given input?
(1) 46 65 97 to managed in 23 be 13 load healthy style work 89 55 34
(2) 46 work 65 style 97 in 23 be 13 load healthy 89 managed 55 to 34
(3) 46 65 97 work style in 23 be 13 load healthy managed to 89 55 34
(4) 34 46 55 65 89 97 work syle in 23 be, 13 load healthy managed to
(5) 89 55 34 work style in 23 be 13 load healthy managed to 46 65 97

4. In which, step are the elements be 13 healthy in found in the


same order?
(1) Third
(2) Sixth
(3) Fourth
(4) The given order of elements is not found in any stop
(5) Fifth

5. How many steps will be required to complete the given


arrangement based on the given input?
(1) Eight
(2) Ten
(3) Seven
(4) Nine
(5) Six

6. Which elements is eight to left of the elements which is twelfth


from the end of the third last step?
(1) 97
(2) 23
(3) Work
(4) Style
(5) To
Directions (Q. Nos. 7-11) The following questions consisting of a question and two statements I and II
given below. You have to decide whether the data given in the statements are sufficient to answer the
questions. Read both the statements and choose the most appropriate option.

Buy Printed Study Material for IBPS, SBI Bank Exams


http://www.bankexamportal.com/study-kit

Page 3

Online Coaching for IBPS, SBI Bank Exams


http://www.bankexamportal.com/elearning

7. All are the five friends F, G, H, I and J, situated in a straight line


facing North?
I. F sits at one of the extreme ends of the line only two people sit between F and H. J sits to immediate left
of H. G and I are immediate neighbours of each other.
II. J faces North and sits at one of the extreme ends of the line. Only two peoples sit between J and I. H sits
second to the right of I. H sits to immediate left of G.
(1) The data in statement II alone are sufficient to answer the question. Which the data one alone are not
sufficient to answer the question.
(2) The data in statement I alone are sufficient to answer the question while the data in statement II alone are
not sufficient, to the answer the question.
(3) The data either in statement I alone or statement II alone are sufficent to answer the question.
(4) The data is even in both statements I and II together are not sufficient to answer the question.
(5) The data in both statements I and II together unnecessary to answer the question.

8. How never will be written in a code language?


I. In that code language never ever go there is written as an ja ni ho.
II. In that code language go there and come back is written as ma ho sa ni da.
(1) The data in statement II alone are sufficient to answer the question, while the data one alone are not
sufficient to answer the question.
(2) The data in statement I alone are sufficient to answer the question while the data in statement II alone are
not sufficient to the answer the question.
(3) The data either in statement I alone or statement II alone are sufficient to answer the question.
(4) The data is even in both statements I and II together are not sufficient to answer the question.
(5) The data in both statements I and II together unncessary to answer the question.

9. Is C the mother of B?
I. P is the father of A and B. R has only the one brother P. C is sister in law of R. R is unmarried F is the
mother of R. F has only two childrens.
II. H has only two childrens P and R. P is father of A. B is the only brother of A. R is unmarried. H is the
father-in-law of C.
(1) The data in statement II alone are sufficient to answer the question, while the data one alone are not
sufficient to answer the question.
(2) The data in statement I alone are sufficient to answer the question while the data in statement II alone are
not sufficient to the answer the question.
(3) The data either in statement I alone or statement II alone are sufficient to answer the question.
(4) The data is even in both statements I and II together are not sufficient to answer the question.
(5) The data in both statements I and II together unnecessary to answer the question.

10. Five movies P, Q, R, S and T are scheduled to be screened from


Monday to Friday of the same week. Which movie will be screened
on Friday?
Buy Printed Study Material for IBPS, SBI Bank Exams
http://www.bankexamportal.com/study-kit

Page 4

Online Coaching for IBPS, SBI Bank Exams


http://www.bankexamportal.com/elearning
I. Movies P will be screened on Tuesday. Only two movies will be screened between movies R and S. R will
be screened before S.
II. Movie Q will be screened on Wednesday. Only one movie will be screened between movies Q and R. S
will be screened immediate before movie T.
(1) The data in statement II alone are sufficient to answer the question, while the data one alone are not
sufficient to answer the question.
(2) The data in statement I alone are sufficient to answer the question while the data in statement II alone are
not sufficient to the answer the question.
(3) The data either in statement I alone or statement II alone are sufficient to answer the question.
(4) The data is even in both statements I and II together are not sufficient to answer the question.
(5) The data in both statement I and II together unnecessary to answer the question.

11. How far is point R from point P?


I. A person starts from point P, walks 15 m to the South takes right turn and walks 4 m. He then takes left
turn and walks 7 m. He takes left turn again, walks for 4 m and reaches point Q. If the percent takes left turn
and walks 7 m, he will reach point R.
II. A person starts from point P, walks 14 m towards the East, takes a left turn and walks 3 m. He than takes
a left turn again and walks for 14 m to reach point N. If a takes a right turn from point N and walks 5 m, he
will be 27 m away from point R.
(1) The data in statement II alone are sufficient to answer the question, while the data one alone are not
sufficient to answer the question.
(2) The data in statement I alone are sufficient to answer the question while the data in statement II alone are
not sufficient to the answer the question.
(3) The data either in statement I alone or statement II alone are sufficient to answer the question.
(4) The data is even in both statements I and II together are not sufficient to answer the question.
(5) The data in both statements I and II together unnecessary to answer the question.
Directions (Q. Nos. 12-17) Study the information carefully and answer the questions.
M, N, O, P, Q, R, S and T are sitting around a circular area of equal distances between each other, but not
necessarily in the same order some of the people are facing the center while some face outside. (i.e., in a
direction opposite to the centre).
Note : Facing the same direction means one faces the centre then the other also faces the centre and viceversas. Facing opposite direction means if one person faces centre then the other faces outside and viceversa.
S sits second to the right of P. P faces the centre. T sits second the left of S. T is an immediate neighbour of
both O and Q. R sits second to the left of Q. M sits seconds to the left of R. T sits second to the left of N. M
sits second to the left of O. T faces the same direction as Q. N sits third to the right of M.

12. Who sits exactly between S and T. When counted from the left
of S?
(1) M
(2) Q
(3) P

Buy Printed Study Material for IBPS, SBI Bank Exams


http://www.bankexamportal.com/study-kit

Page 5

Online Coaching for IBPS, SBI Bank Exams


http://www.bankexamportal.com/elearning
(4) O
(5) N

13. How many people in the given arrangement face the centre?
(1) One
(2) Three
(3) Five
(4) Two
(5) Four

14. Who sits second to the right of N?


(1) R
(2) P
(3) T
(4) S
(5) O

15. Four of the following five are alike in a certain way based on the
given sitting arrangement and so form a group. Which is the one
that does not belong to that group?
(1) R
(2) S
(3) Q
(4) O
(5) M

16. Which of the following is true regarding T as per the given


sitting arrangements?
(1) T faces the centre.
(2) Only two people sit between N and T.
(3) Only three people sit between T and P.
(4) S sits second to the left of T.
(5) Only one persons sits between T and R.

17. What is Ms position with respect to T?


(1) Second to the left
(2) Fifth to the right
(3) Third to the right
(4) Third to the left
(5) Fourth to the right

Buy Printed Study Material for IBPS, SBI Bank Exams


http://www.bankexamportal.com/study-kit

Page 6

Online Coaching for IBPS, SBI Bank Exams


http://www.bankexamportal.com/elearning
Directions (Q. Nos. 18-23) In these questions two/three statements followed by two conclusions
numbered I and II have been given. You have to take the given statements to be true even if they seem
to be at variance from the commonly known facts and then decide which of the given conclusions
logically follows the statements disregarding commonly known facts.

18. Statements
Some prices are costs.
Some costs are amounts.
All amounts are expenses.
Conclusions
I. At least some amounts are prices.
II. All amounts being prices is a possibility.
(1) Either conclusion I or II is true
(2) Both conclusion I and II are true
(3) Neither conclusion I nor II is true
(4) Only conclusion II is true
(5) Only conclusion I is true

19. Statements
All invitations are rejections.
Some invitations are celebrations.
No rejection is an attraction.
Conclusions
I. Some celebrations are rejections.
II. All celebrations are rejections.
(1) Either conclusion I or II is true
(2) Both conclusion I and II are true
(3) Neither conclusion I nor II is true
(4) Only conclusion II is true
(5) Only conclusion I is true

20. Statements
All grades are scales.
All scales are categories.
Conclusions
I. All grades are categories.
II. All categories are scales.
(1) Either conclusion I or II is true

Buy Printed Study Material for IBPS, SBI Bank Exams


http://www.bankexamportal.com/study-kit

Page 7

Online Coaching for IBPS, SBI Bank Exams


http://www.bankexamportal.com/elearning
(2) Both conclusion I and II are true
(3) Neither conclusion I nor II is true
(4) Only conclusion II is true
(5) Only conclusion I is true

21. Statements
Some metals are papers.
All papers are alloys.
No alloy is a wood.
Conclusions
I. All woods being metals is a possibility
II. All metals being woods is a possibility.
(1) Either conclusion I or II is true
(2) Both conclusion I and II are true
(3) Neither conclusion I nor II is true
(4) Only conclusion II is true
(5) Only conclusion I is true

22. Statements
Some metals are papers.
All papers are alloys.
No alloy is a wood.
Conclusions
I. No paper is a wood.
II. At least some meals are alloys.
(1) Either conclusion I or II is true
(2) Both conclusion I and II are true
(3) Neither conclusion I nor II is true
(4) Only conclusion II is true
(5) Only conclusion I is true

23. Statements
Some prices are costs.
Some costs are amounts.
All amounts are expenses.
Conclusions
I. No expense is a cost.
II. At least some expenses are prices.

Buy Printed Study Material for IBPS, SBI Bank Exams


http://www.bankexamportal.com/study-kit

Page 8

Online Coaching for IBPS, SBI Bank Exams


http://www.bankexamportal.com/elearning
(1) Either conclusion I or II is true
(2) Both conclusion I and II are true
(3) Neither conclusion I nor II is true
(4) Only conclusion II is true
(5) Only conclusion I is true

24. This question consist of a statements followed by two course of


actions numbered I and II given below it. A course of action is an
administrative decision to be taken for improvements, follow-up or
further action in the regard to the problems, policy etc. You have to
assumed everything in the statement to be true and then decide
which of the suggested courses of actions logically follows from the
given statements.
Statements
There have been structural shifts from agriculture to industry in country Z. While industries contribute to
67% of the countrys GDP. It is leading to a study and undesirable declined in the participation of labour
force in agriculture.
I. The government should curtail number of industries borrowing up in the country in order to promote the
agricultures.
II. Incentives for working in agriculture should be made competitive with that of other sectors.
(1) Either conclusion I or II is true
(2) Both conclusion I and II are true
(3) Neither conclusion I nor II is true
(4) Only conclusion II is true
(5) Only conclusion I is true
Directions (Q. Nos. 25-27) Study the following information carefully and answer the questions.
Each of the six people U, V, W, X, Y and Z stayed for a different number of day in India. X stayed for less
number of days then only V, Y stayed for more number of days then only two people. U and did not stay
less than Y. W did not stay for the minimum number of days. The one who stayed for the second lowest
number of days stayed for eight days. U stayed for 17 days in India.

25. Which of the following is possibly the number of days for which
it stayed in India?
(1) 5
(2) 32
(3) 23
(4) 20
(5) 17

Buy Printed Study Material for IBPS, SBI Bank Exams


http://www.bankexamportal.com/study-kit

Page 9

Online Coaching for IBPS, SBI Bank Exams


http://www.bankexamportal.com/elearning

26. Which of the following is true with respect to Z as per the given
information?
(1) The possible number of days for which Z stayed in India is 14 days
(2) Z stayed for the minimum number of days
(3) None of the given options is true
(4) Only two peoples stayed for more number of days than Z
(5) Z definitely stayed for more number of days than Y

27. Which of the following may be the possible number of days for
which Y stayed in India?
(1) 11
(2) 8
(3) 19
(4) 5
(5) 25
Directions (Q. Nos. 28-32) Study the information carefully and answer the given questions.
Ten people are sitting in two parallel rows containing five people each in such a way that there is an equal
distance between adjacent persons in row- I J, K, L, M and N are seated and all of them are facing North and
in row-II P, Q, R, S and T are seated and all facing South (but not necessarily in the same order). Each
person also likes different flowers namely Gazania, Plumeria, Tulip, Orchids, Lily, Rose, Daffodil, Daisy
Peony and Dahila (but not necessarily in the same order).
N sits exactly in the centre of the row and faces the one who likes orchids. Only one person sits between N
and the one who likes Lily. S faces one of the immediate neighbour of the one who likes Lily. Only one
person sits between P and the one who likes Orchids. P is not an immediate neighbour of S. P faces one of
the immediate neighbour of L. M is neither an immediate neighbours of L nor faces S. Q is not an immeidate
neighbour of A and faces the one who likes Plumeria. M does not face R. M faces one of the immediate
neighbour of the one who likes rose. Only one person sits between the one who likes Rose and the one who
likes Gazania. The one who likes Peony and the one who likes Daisy face each other. Only two people sit
between the one who likes Daisy and the One who likes Daffodil. J does not like Daffodil. The one who
likes Dahlia sits fourth to the right of the one who likes Peony.

28. Who amongst the following is a immediate neighbour of the one


who likes Peony?
(1) Who likes Rose
(2) Who likes Lily
(3) Who likes Gazania
(4) K
(5) T

Buy Printed Study Material for IBPS, SBI Bank Exams


http://www.bankexamportal.com/study-kit

Page 10

Online Coaching for IBPS, SBI Bank Exams


http://www.bankexamportal.com/elearning

29. Which of the following is definitely true as per the given


arrangements?
(1) Lily
(2) Gazania
(3) Rose
(4) Orchids
(5) Daffodil

30. Which of the following flowers does S like?


(1) L face R
(2) Q is an immediate neighbour of P
(3) The one who likes Daisy is an immediate neighbour of P
(4) Q likes Daffodil
(5) The one who likes Dahlia faces N

31. Which of the following pairs represent the people sitting at the
extreme end of the two rows?
(1) P, J
(2) Q, L
(3) R, K
(4) S, K
(5) S, L

32. Who amongst the following likes Tulip?


(1) N
(2) P
(3) S
(4) Q
(5) J

33. Read the following information carefully and answer the


question which follows :
As per a recent survey 85% of the world force today is either
actively looking for a job or open to talking to recruiters and
relevant opportunities, the ones who are satisfied with their job.

Buy Printed Study Material for IBPS, SBI Bank Exams


http://www.bankexamportal.com/study-kit

Page 11

Online Coaching for IBPS, SBI Bank Exams


http://www.bankexamportal.com/elearning

Which of the following may be a reason for the above mentioned


results of the survey?
(1) As there is the wide array of opportunity today and head hunters are always seeking for professionals,
people are not willing to let go of an opportunity to do something new and challenging.
(2) As per a recent survey more than 40% people today are dissatisfied with their jobs as there is the wide
gap between what they study in the class room as compared to the actual work they do
(3) Some people in the country take education loan for completing higher education and are expected to
repay the loans themselves after they start working
(4) As employees today are of different age group and back grounds it becomes difficult for the HR
department to organise employment engagement activities to improve the job satisfaction of employees
(5) Only a few people study for high specialised coursed in the country and they are always in demand as
there is high scale gap in many organisations
Directions (Q. Nos. 34-35) The following questions consist of a statements followed by two statements
numbered I and II given below it. You have to decide which of the given statements numbered I and II
weakens or strengthens the statement and mark the appropriate answer.

34. Statement
The world should move towards using bio-fuels in the future as these are more environment friendly as
compared to regular fossil-fuels.
I. While bio-fuels cleaner to burn, the processed to produce the fuel, including the necessary machinery
leads to very high carbon emission.
II. As two-fuels are manufactured from materials such as crop waste, manure and other by products, these,
unlike fossil-fuels, are easily renewable.
(1) Statement I weakens the information while statements II is a natural statement.
(2) Both statement I and II weakens the information.
(3) Statement I strengthens the information while statement II weakens the statement.
(4) Statement I weakens the information while statement II strengths the statement.
(5) Both statements I and II strengthens the information.

35. Statement
Always remain in an air conditioned environment for better health and well-being.
I. Filters in the air-conditioners lower exposure to allergy creating pollen and other outdoor allergies.
II. Spending two much times in an air-conditioned environment result in gradual intolerance of body
towards natural temperatures.
(1) Statement I weakens the information while statements II is a natural statement.
(2) Both statement I and II weakens the information.
(3) Statement I strengthens the information while statement II weakens the statement.
(4) Statement I weakens the information while statement II strengths the statement.
(5) Both statements I and II strengthens the information.
Directions (Q. Nos. 36-14) The following questions have a statement followed by two conclusions (I and II).
Assuming the given statements to be true, find which conclusion is definitely true.

Buy Printed Study Material for IBPS, SBI Bank Exams


http://www.bankexamportal.com/study-kit

Page 12

Online Coaching for IBPS, SBI Bank Exams


http://www.bankexamportal.com/elearning

36. Statement
BA=N> KS
Conclusions
I. A > S II. B K
(1) Either conclusion I or II is true
(2) Both conclusion I and II are true
(3) Neither conclusion I nor II is true
(4) Only conclusion II is true
(5) Only conclusion I is true

37. Statement
BA=N> KS
Conclusions
I. B N II. S < N
(1) Either conclusion I or II is true
(2) Both conclusion I and II are true
(3) Neither conclusion I nor II is true
(4) Only conclusion II is true
(5) Only conclusion I is true

38. Statement
K<=MN=OP
Conclusions
I. L > O
II. L = O
(1) Either conclusion I or II is true
(2) Both conclusion I and II are true
(3) Neither conclusion I nor II is true
(4) Only conclusion II is true
(5) Only conclusion I is true

39. Statement
KL=MN=OP
Conclusions
I. K < P II. M O
(1) Either conclusion I or II is true
(2) Both conclusion I and II are true
(3) Neither conclusion I nor II is true
(4) Only conclusion II is true
(5) Only conclusion I is true

Buy Printed Study Material for IBPS, SBI Bank Exams


http://www.bankexamportal.com/study-kit

Page 13

Online Coaching for IBPS, SBI Bank Exams


http://www.bankexamportal.com/elearning

40. Statement
X<YZ=W
KJ>Z
Conclusions
I. Z < K II. Y W
(1) Either conclusion I or II is true
(2) Both conclusion I and II are true
(3) Neither conclusion I nor II is true
(4) Only conclusion II is true
(5) Only conclusion I is true

41. Statement
X<YZ=W
KJ>Z
Conclusions
I. X J II. K > Y
(1) Either conclusion I or II is true
(2) Both conclusion I and II are true
(3) Neither conclusion I nor II is true
(4) Only conclusion II is true
(5) Only conclusion I is true
Directions (Q. Nos. 42-44) Study the following information and answer the given questions. A and Y
are brothers of K. Y is the son of P and S. P is the daughter of X. M is the father in law of S. Q is the
son of X.

42. If J is brother of X, then how is J related to Q?


(1) Uncle
(2) Nephew
(3) Cannot be determined
(4) Brother-in-law
(5) Son-in-law

43. How is Y related to M?


(1) Nephews
(2) Father
(3) Brother-in-law
(4) Grandson
(5) Brother

44. How is K related to Q?


Buy Printed Study Material for IBPS, SBI Bank Exams
http://www.bankexamportal.com/study-kit

Page 14

Online Coaching for IBPS, SBI Bank Exams


http://www.bankexamportal.com/elearning
(1) Cannot be determined
(2) Niece
(3) Daughter (4) Nephew
(5) Son in law

45. Read the given information carefully and answer the question.
Five fisherman from country X were sentenced to imprisonment by
the High Court of country Y on charges of smuggling narcotics
from country X to country Y. Which of the following supports the
legal action taken by country Y?
A. Country X had given details of a new fisherman that had no record of involvement in drug related
activities.
B. The navel authorities of both the country X and Y have found traces of narcotics in the fish boxes that
were shipped from country X to country Y by these fisherman.
C. All the fish boxes that are shipped between countries undergo a strict process of checking.
D. It was found that two of the five fisherman who were sentenced to death penalty were earlier imprisoned
for the same reason a few years back.
(1) B and C
(2) B and E
(3) Only D
(4) None of these
(5) Only B

Directions (Q. Nos. 46-50) Read the given information and answer the given questions.
Eight people A, B, C, D, E, F, G and H live on separate floors of an 8-floor building. Ground floor is
numbered H, first floor is numbered 2 and so on until the top most floor is numbered 8.
C lives on floor number 3. Only one person lives between C and F. Only one person lives between C and G.
E lives immediately above B
H lives immediately above F. H lives on any of the floors below D.
Only one person lives between D and A. D lives above A.

46. Who lives on the floor immediately below D?


(1) Other than those given as options
(2) H
(3) A

Buy Printed Study Material for IBPS, SBI Bank Exams


http://www.bankexamportal.com/study-kit

Page 15

Online Coaching for IBPS, SBI Bank Exams


http://www.bankexamportal.com/elearning
(4) D
(5) C

47. Which of the following is true with respect to the given


information?
(1) Only one person lives between E and G
(2) A lives on an even numbered floor
(3) A lives immediately below C
(4) F lives on floor number 5
(5) A lives immediately above H

48. How many people live between the floors on which D and C,
live?
(1) One
(2) More than three
(3) Three (4) Two
(5) No more

49. Who amongst the following lives between B and G?


(1) No one
(2) C
(3) E
(4) H
(5) D

50. Who amongst the following lives on floor number 5?


(1) A
(2) G
(3) F
(4) D
(5) H

Buy Printed Study Material for IBPS, SBI Bank Exams


http://www.bankexamportal.com/study-kit

Page 16

Online Coaching for IBPS, SBI Bank Exams


http://www.bankexamportal.com/elearning

English
Directions (Q. Nos. 51-55) In the following questions, the sentences have two blank, each blank
indicates that something has been confined. Choose the word that best fits in the meaning of the
sentence as a whole.

51. .............. Dinosaurs live to be the ancestors of modern birds, the


discovery of soft tissue in Dinosaurs fossils containing proteins
found only in birds is very ...........
(1) Though, unfortunate
(2) Recently, common
(3) Since, exciting
(4) Perhaps, remarkable
(5) Moreover, welcome

52. The van Allen belts which are ............ between 13000 to 60000
km above the Earths surface were discovered in 1958 and contain
electrons which play ............ with electronic systems on satellites.
(1) Somewhere, damages
(2) located, havoc
(3) Position, mayhem
(4) Situated, part
(5) Vicinity, confusion

53. Rice farmers use ........... methods of farming and old equipment
making the cost of price ......... twice as much as it is in other
countries.
(1) Obsolete, economical
(2) Modern, expensive
(3) Future, virtuality
(4) Antiquated, almost
(5) Sustainable, approximate

54. Some experts believe that the best .......... to reduce the number
of accidents caused by drivers using their mobile phones to create
an app/ ......... communications when the vehicle in is motion and
unblocking them when the vehicle reaches its destination.
Buy Printed Study Material for IBPS, SBI Bank Exams
http://www.bankexamportal.com/study-kit

Page 17

Online Coaching for IBPS, SBI Bank Exams


http://www.bankexamportal.com/elearning
(1) Tactic, block
(2) Strategy, disabiling
(3) Part, facilitating
(4) Decision, expediting
(5) Method, further

55. Services provided by money transfer companies are ............... for


people who dont use bank account or who are working for from
home and are helpful to ........... trade and dispersion wealth.
(1) Fundamental, compute
(2) Prerequisite, invigorating
(3) Vitally, strengthen
(4) Essentally, boester
(5) inherent, foster
Directions (Q. Nos. 56-60) Rearrange the given six sentences A, B, C, D, E and F in a proper sequence
so as to form a meaningful paragraph and then answer the given questions.
A. She follows the foot steps of other renowned female politicians like Chiles Michelle Bachelet and
Germanys Angela Merkel.
B. Women are also rein to the fore front in other parts of the government.
C. And this leadership can be seen in the growing number of women who are becoming political leaders and
the most recent being Dilma Rouseff who looks over as Brazils first women President.
D. It is defined as the ability to influence or lead through persuasion or attraction by co-opting people rather
than coercing them.
E. The leadership of women in politics business and society is becoming evident across the globe.
F. These trends represent the growing needs for soft power in todays world.

56. Which of the following should be the FOURTH sentence after


the rearrangement?
(1) A
(2) B
(3) C
(4) F
(5) E

57. Which of the following should be the SECOND sentence after


the rearrangement?
(1) C
(2) B
(3) F

Buy Printed Study Material for IBPS, SBI Bank Exams


http://www.bankexamportal.com/study-kit

Page 18

Online Coaching for IBPS, SBI Bank Exams


http://www.bankexamportal.com/elearning
(4) D
(5) E

58. Which of the following should be the SIXTH (LAST) sentence


after the rearrangement?
(1) E
(2) D
(3) B
(4) A
(5) F

59. Which of the following should be the FIFTH sentence after the
rearrangement?
(1) E
(2) D
(3) B
(4) F
(5) C

60. Which of the following should be the FIRST sentence after the
rearrangement?
(1) A
(2) C
(3) B
(4) F
(5) E
Directions (Q. Nos. 61-70) Read the following passage carefully and answer the questions. Certain
words/phrases are given in bold to help you locate them while answering some of the questions.
Infrastructure projects take a long time to built but then deliver cash flows over an extended period, precision
funds have liabilities thats stretch our several decades. Why not get the letter to finance the former? A new
report from the Organisation for Economic Cooperation and Development (OECD) estimates that global
pension funds have just point 9% of their portfolios in pure infrastructure in part i.e., due to the OECDs
decision to define infrastructure assets as untrusted debt and equity.
But pension funds has significant exposure to tested shares and bonds of power companies and the due from
the point of view public policy however, the OECDs definition is the correct one. The utility shares owned
by pension funds are those of power companies that were privatised in the 1980s and 1990s and the
infrastructure they operate is a result of government spending in previous decade. Governments would rather
not see more infrastructure get built, thought at the moment public financial are very tight. They would rather
not bear the whole burden. The difficult bit about infrastructure projects apart from the original decision to

Buy Printed Study Material for IBPS, SBI Bank Exams


http://www.bankexamportal.com/study-kit

Page 19

Online Coaching for IBPS, SBI Bank Exams


http://www.bankexamportal.com/elearning
commission them is the cost of construction i.e., where governments would like pension funds and the rest of
the private sectors to open their wallets.
Risk is clearly one important factor. Pension funds want reliable cash flows that can be used to pay retres, not
the uncertains that are associated with projects. As the OECD points out there is a lack of objective high
quality data on infrastructure investments. These make this difficults for funds to calculate how infrasture
would fit in to there portfolio. Another problems is that pension funds may lack the expertise to get involved
in such large projects, they have to invest via and infrastructure funds and pay a management fee for the
previlege. The biggest infrastructure investors so far have been the joined Australian and Canadian pension
funds which can benefits from economics of scale. Britain is trying to achieve the same effect by setting up
pension investment platform which will pool infrastructure investment. However, the scheme is slow to get
going and not be sufficient fund brattish highest profile projects, a proposed high speed rail line additional
problems include the lack of political certainty. Capital spending is often the first item to be cut when
governments run into budget difficulties and tough decision and put of to suit electoral cycles. One possible
solution is for governments to borrow separate sum to finance infrastructure spending with the stated intension
of the selling assets to the likes of pension funds over a number of years such are debt would be recorded
separately in the National Accounts. An ultimative option could be a National Investment Bank among the
lines of European investment bank. It would borrow from the market and used its capital to guarantee the
equity portion of the infrastructure project. They would allow pension funds to buy the more secure debt
elements of project findings the needs is clear among the G-7 countries. Only Italy is recorded as having
worlds infrastructure. There is no shortage of protection funding-Britains pension assets are equal to 112% of
GDP. Clearly to can be put together. The Olympics showed that Britain can built projects on time when the
country puts its mind to it.

61. Which of the following is the role of pensions investment


platform?
(1) To create guidelines regarding infrastructure investment in retain.
(2) To provide services only to small pension funds to invest in infrastructure.
(3) To monitor prestigious infrastructure projects and public private partnership
(4) To provide guidance to authorities in Europe regarding infrastructure development.
(5) To garner funding for infrastructure projects.

62. What is the authors view of OECD report on infrastructure?


(1) He feels that the report unnecessarily focusses on the negative aspects of investing in government
projects.
(2) He is an agreement with the data and contentions cited in the report.
(3) He is in favour of the OECDs view of governments keeping investment in infrastructure on hold.
(4) He opposes the OECDs restrictions on diverting funds to infrastructure development.
(5) He recommends that investments levels prescribed by the report beared opted by Britain Accordingly

63. According to the passage, why is there a need for Britain to


invest in infrastructure?

Buy Printed Study Material for IBPS, SBI Bank Exams


http://www.bankexamportal.com/study-kit

Page 20

Printed Study Material, for Bank, SBI, IBPS - PO, Clerk Exams
IBPS Specialist Officer Exam Study Material - 100% Syllabus Covered
http://bankexamportal.com/study-kit/ibps-specialist-officer
Printed Study Material for SBI PO Exam 2015 - 100% Syllabus

Covered
http://bankexamportal.com/study-kit/sbi-po
Printed Study Materials for IBPS-PO (Probationary Officer) Exam
http://bankexamportal.com/study-kit/ibps-po
IBPS Clerk Study Kit
http://bankexamportal.com/study-kit/ibps-clerk
Essay Writing Skills Improvement Programme (EWSIP)
http://iasexamportal.com/civilservices/study-kit/essay-mains

For More Information Click Given below link:


http://bankexamportal.com/study-kit

Online Coaching for IBPS, SBI Bank Exams


http://www.bankexamportal.com/elearning
(1) Britains desire to surpass Italy in infrastructure development
(2) Surplus pension funds need to be gainfully diverted to avoid misuse
(3) Infrastructure is deficient and in need of an overhaul
(4) Britains prestige as host of the Olympics is at stake
(5) Not clearly mentioned in the passage

64. Which of the following factor(s) impact(s) investment decisions


of the pension funds?
A. Political uncertainty and financial risk.
B. Requisite expertise in infrastructure projects.
C. Returns on investment.
(1) Only B
(2) A and B
(3) Only A
(4) All of these
(5) B and C

65. Which of the following is the main objective of the author in


writing the passage?
(1) To examine the misappropriation of Britains pension funds
(2) To estimate how unprofitable it is for the country if pension funds are used to create infrastructure
(3) To urge the government to reduce risky exposure of pension funds and bonds an invest in safe options
(4) To analyse the claims made by the OECD report and point out inconsistencies
(5) To suggest that pension funds can and should more in infrastructure

66. Choose the word which is opposite in meaning to the word


Extended given in bold as used in the passage.
(1) Absolute
(2) Fleeting
(3) Equitable
(4) Steady
(5) Regular

67. Choose the word which is most nearly the same in meaning to
the word Commission given in bold as used in the passage.
(1) Agency
(2) Committee
(3) Contract
(4) Payment
(5) Expense

Buy Printed Study Material for IBPS, SBI Bank Exams


http://www.bankexamportal.com/study-kit

Page 21

Online Coaching for IBPS, SBI Bank Exams


http://www.bankexamportal.com/elearning

68. Which of the following can be inferred in the content of the


passage?
(1) Britain is making efforts to reform its infrastructure sector
(2) Britains GDP is abysmal and prestigious infrastructure projects should be kept on hold till the pension
and infrastructure projects are reformed
(3) Investments banks are risky as they are governed by political powers
(4) Britains national accounts data lacks crucial parameters
(5) All the given statements can be inferred in the context of the passage

69. What does the author want to convey through the phrase They
would rather not bear the whole burden?
(1) Government would prefer to privatise utilities like power etc
(2) Government are vary about the public takeout of pension funds going bust
(3) Governments short on capital, are cautious about society financing infrastructure as it is costly
(4) The private-sector is unwilling to continue their monetary support of loss making public facilities
(5) Other than those given as options

70. According to the passage, which of the following can be said


about infrastructure investment?
A. It provides tremendous opportunities to investors as there is a huge need for infrastructure.
B. Creative solutions are required to generate the necessary investment.
C. It is crucial for governments to invest in infrastructure to win elections.
(1) None
(2) A and B
(3) Only A
(4) Only B
(5) A and C
Directions (Q. Nos. 71-75) Which of the phrase given against the sentence should replace the word/
phrase given in bold in sentence to make it grammatically correct? If the sentence is given as it is
given and no correction is required, mark No correction required as the answer.

71. Getting tax refunds is a problematic not only for the common
man but also sensor officials.
(1) are problems (2) is a problem
(3) are a problem (4) is problem
(5) no correction required

Buy Printed Study Material for IBPS, SBI Bank Exams


http://www.bankexamportal.com/study-kit

Page 22

Online Coaching for IBPS, SBI Bank Exams


http://www.bankexamportal.com/elearning

72. The World Bank represents a particularly difficult case of


organisation cultural changes.
(1) Represent (2) Representing
(3) Representation
(4) Have representing
(5) No correction required

73. Big companies are hiking stipends for summer internships by


40%, sending out a strong signal. That they expects the economy to
new up.
(1) Them expecting
(2) They expect
(3) These expectation
(4) Expect these
(5) No correction required

74. Declining crude prices have raise hopes of interest of rate cuts.
(1) Have rise
(2) Have rising
(3) Have raised
(4) Has risen
(5) No correction required

75. India continues to be on the top of the global consumer


confidence under this year as well.
(1) Continue to being
(2) Continuing to be
(3) Continuing too be
(4) Continue to be
(5) No correction required
Directions (Q. Nos. 76-80) Read this sentence to find out whether there is any grammatical mistakes/
error in it. The error, if any, will be in one part of the sentence. Mark the part with the error as your
answer. If there is no error, mark No error as your answer. (Ignore the errors of punctuation, if any)

76. After the cash-strapped corporation/expressed its inability to


provide funds/for various devlop- mental works, the office bearers
of the civic body started/lobbying at special financial and from State
government.
Buy Printed Study Material for IBPS, SBI Bank Exams
http://www.bankexamportal.com/study-kit

Page 23

Online Coaching for IBPS, SBI Bank Exams


http://www.bankexamportal.com/elearning
(1) After the cash-strapped corporation
(2) Expressed its inability to provide funds
(3) For various development works, the office bearers of the civic body started
(4) Lobbying for special financial and from State government
(5) No error

77. Three things are/essential for a start up to succeed,/a product


that is in demand,/a good team and as well as a minimum
expenditure.
(1) Three things are
(2) Essential for a start up to succeed
(3) A product that is in demand
(4) A good team as well as a minimum expenditure
(5) No error

78. We all have at/some point in our lives/wondered if we are


actually/being too nice to someone.
(1) We all have at
(2) Some point in our lives
(3) Wondered if we are actually
(4) Being too nice to someone
(5) No error

79. Anticipating a foggy winter,/the railway authorities has/decided


to/cancel thirty trains.
(1) Anticipating a foggy winter
(2) The railway authorities have
(3) Decided to
(4) Cancel thirty trains
(5) No error

80. New businesses today/are increasing by focusing on/introducing


various perks and motivation programmes/to help retain
employees.
(1) New businesses today
(2) Are increasing focusing on
(3) Introducing various perks and motivation programmes
(4) To help retain employees
(5) No error

Buy Printed Study Material for IBPS, SBI Bank Exams


http://www.bankexamportal.com/study-kit

Page 24

Online Coaching for IBPS, SBI Bank Exams


http://www.bankexamportal.com/elearning
Directions (Q. Nos. 81-90) Read the following passage carefully and answer the given questions.
Certain words are given in bold to help you to locate them while answering some of the questions.
In our day and age, technology is omnipresent and integral part of our lives. However, although, the main
purpose of technology is to make our lives easier, the reactions on opinions and technology are very diverse.
This year, various sessions at the Womens Forum covered the influence of new technologies in our daily life.
It is worthwhile analysing two contrasting perspectives in depth, to understand how broad this debate is. On
the one hand, technology and digital media can be a great help to reach out to other people and spread your
message to a very large platform. The session How to be a digital influencer was analysing exactly this
question, and introduced various platforms and strategies on how too use the digital world to your advantage.
Social media removes all distances : Geographic, social and hierarchical. You cna reach, at any time, and
communication in real time. Thus, a permanent link for communication has been created. Today; everyone
has the ability to transmit knowledge and thus credibility of the creator of knowledge is more important than
even before. Technology has taken the universal communication method of storytelling and transformed it, in
a way that now more stories can reach more people through social media platforms than ever before. People
should find their digital media voice, tell sharable stories and drive engagement. Also, it is very important to
be authentic and truly about the message you are trying to convey through the digital media. Another session,
the very same day, analysed how exactly the opposite is true and how new technologies actually cause stress
and suffering, because of a concept referred to as infobesity, we have changed the way we consume and
generate information. People are trying to live ten lives at once and a lot of stress and anxiety occurs from
that. Thus, we are suffering from digital bulimia, meaning that we take in a lot of information at once, without
really processing if for ourselves, and in turn create a lot of new information. We are at the same time creators
and victims of information overload. Thus, it is obvious that technological advancement has failed at its
mission of making everybodys life easier, as many people are reacting negatively to it.

81. According to the passage, which of the following is not true


about the characteristics of infobestiy?
(1) The pace of our lives has increased significantly due to technology
(2) With the help of technology, people try to domultiple things at the same time, leading to stress
(3) Technology makes our lives sedentary and leads to health problems
(4) Due to technology, each individual is bombarded with excess information
(5) All the options are true according to the passage

82. Which of the following is most opposite in meaning of the word


given as bold as used in the passage?
(1) Similar
(2) Separate
(3) Interwoven
(4) Simultaneous
(5) Together

83. Which of the following can be said about the invention of the
car?
Buy Printed Study Material for IBPS, SBI Bank Exams
http://www.bankexamportal.com/study-kit

Page 25

Online Coaching for IBPS, SBI Bank Exams


http://www.bankexamportal.com/elearning
A. More than being a boon, this invention has become an inconvenience to us.
B. It has invoked in us the desire to travel to places that we would not have otherwise gone to
C. It has enabled us to have more time for ourselves
(1) A and C
(2) B and C
(3) A and B
(4) All of these
(5) Only B

84. Which of the following can be the most suitable title for the
passage?
(1) The paradox of technological advance
(2) The different types of technology available
(3) Technology propagation-the road to nowhere
(4) Technology and gainful insight
(5) Technological progress across the continents

85. According to the author, which of the following can be said


about reaching to people through the medium of technology?
A. Todays digital age has made it essential for an individual to send a message that is genuine in nature.
B. The ability to impact many people through a message is solely dependent on the number of times the
same message goes out, regardless of its authenticity.
C. Technology has made it easier to reach out to masses of people at the same time.
(1) Only C
(2) All of these
(3) A and C
(4) A and B
(5) Only B

86. Which of the following is true according to the passage?


A. It is in our own interest that we keep check on the amount of digital information that we generate.
B. The promise on which technology was designed was that it should make our lives simpler.
C. It is best that we learn to survive without any technological intervention in our lives.
(1) A and B
(2) All of these
(3) B and C
(4) Only A
(5) Only B

Buy Printed Study Material for IBPS, SBI Bank Exams


http://www.bankexamportal.com/study-kit

Page 26

Online Coaching for IBPS, SBI Bank Exams


http://www.bankexamportal.com/elearning

87. According to the passage, which of the following is authors


opinion/statement about technology?
(1) Beware! Technology can only bring doom if allowed to proliferate.
(2) Take responsibility for how you let technology impact your life.
(3) Go head and invest in the latest technology, it is worth it.
(4) What the future of technology holds for us is unknown, wait and watch
(5) Other than those given as options.

88. Which of the following is most nearly the same in meaning to the
word given in bold as used in the passage? Engagement
(1) Appointment
(2) Involvement
(3) Meeting
(4) Rendezvous
(5) Date

89. Which of the following is most nearly the same in meaning to the
word given in bold as used in the passage?Credibility
(1) Calculated
(2) Deliberated
(3) Believed
(4) Created
(5) Patterned

90. Which of the following is the most opposite in meaning to the


word given in bold as used in the passage? Spread
(1) Collect
(2) Take
(3) Restrict
(4) Join
(5) Multiply
Directions (Q. Nos. 91-100) In the given passage, there are blanks, each of which has been numbered.
Against each, five words are suggested, one of which fits the blank appropriately. Find the
appropriate word in each case.
Changing an organisations culture is one of the most difficult leadership challenges. Thats because an
organisations culture (91) interlocking set of goals, roles, processes, values communications parctices,
attitudes and assumptions. These elements (92) together as a mutually reinforcing system and combine to

Buy Printed Study Material for IBPS, SBI Bank Exams


http://www.bankexamportal.com/study-kit

Page 27

Online Coaching for IBPS, SBI Bank Exams


http://www.bankexamportal.com/elearning
prevent any (93) to change it. That is why single-fix challenges, such as the introduction of teams. Of lean,
or agile, or scrum or knowledge management or some new process, may (94) to make progress for a while,
but (95) the interlocking elements of the organisational culture (96) over and the change is inexorably drawn
(97) into the existing organisational culture. Changing a culture is large scale undertaking and eventually all
of the organisational focus for changing minds will need to be put in play. However, the (98) in which they
are deployed has a Critical (99) on the (100) of success.

91.
(1) Less
(2) Embraces
(3) Comprises
(4) Makes
(5) Composes

92.
(1) Amalgamate
(2) Conjoin
(3) Fill
(4) Stand (5) Attach

93.
(1) Attempt
(2) Energy
(3) Ways
(4) Power
(5) Tries

94.
(1) Look
(2) Appear
(3) Perform
(4) Suspect
(5) Seek

95.
(1) Especially
(2) Presently
(3) Likely
(4) Eventually
(5) Lately

Buy Printed Study Material for IBPS, SBI Bank Exams


http://www.bankexamportal.com/study-kit

Page 28

Online Coaching for IBPS, SBI Bank Exams


http://www.bankexamportal.com/elearning

96.
(1) Bounce
(2) Throw
(3) Seize
(4) Jump
(5) Take

97.
(1) Forth
(2) Ahead
(3) Gain
(4) Set
(5) Back

98
(1) Order
(2) Succession
(3) Idea
(4) Step
(5) Label

99.
(1) Affect
(2) Impact
(3) Control
(4) Jolt
(5) Shake

100.
(1) Change
(2) Odd
(3) Risk
(4) Likelihood
(5) Option

Buy Printed Study Material for IBPS, SBI Bank Exams


http://www.bankexamportal.com/study-kit

Page 29

Online Coaching for IBPS, SBI Bank Exams


http://www.bankexamportal.com/elearning

Data Analysis and Intelligence


Directions (Q. Nos. 101-106) In these questions a number of series is given. After the series a number
is given followed by (a), (b), (c), (d) and (e). You have to complete the series starting with the given
numbers following the sequence of original series and answer the questions that follows the series.

101. 48 24 36 90 315 1417.5


20 (a) (b) (c) (d) (e)
What will come in place of (d)?
(1) 131.25
(2) 133.75
(3) 136
(4) 140
(5) 142.25

102. 2 11 52 183 430 555


4 (a) (b) (c) (d) (e)
What will come in place of (b)?
(1) 92
(2) 98
(3) 82
(4) 88
(5) 96

103. 7 9 21 69 273 1371


6 (a) (b) (c) (d) (e)
What will come in place of (b)?
(1) 19
(2) 22
(3) 18
(4) 20
(5) 17

104. 9 8 12 27 92 435
7 (a) (b) (c) (d) (e)
What will come in place of (c)?

Buy Printed Study Material for IBPS, SBI Bank Exams


http://www.bankexamportal.com/study-kit

Page 30

Online Coaching for IBPS, SBI Bank Exams


http://www.bankexamportal.com/elearning
(1) 19
(2) 12
(3) 13
(4) 15
(5) 16

105. 8 9 21 68 279 1404


5 (a) (b) (c) (d) (e)
What will come in place of (d)?
(1) 184
(2) 207
(3) 212
(4) 196
(5) 228

106. 4 7 19 73 381 2161


8 (a) (b) (c) (d) (e)
What will come in place of (c)?
(1) 106
(2) 169
(3) 156
(4) 184
(5) 174
Directions (Q. Nos. 107-112) Study the following table to answer this questions.

Buy Printed Study Material for IBPS, SBI Bank Exams


http://www.bankexamportal.com/study-kit

Page 31

Online Coaching for IBPS, SBI Bank Exams


http://www.bankexamportal.com/elearning

107. Total number of female employees (manager and officers) in


procurement department is by what percent more than their male
counter part?
(1)9%
(2)79/7 %
(3)45/4%
(4)104/11%
(5)100/11%

108. What is the difference between total number of male officers in


Advertising and Public Relations Departments and the total
number of female managers in these two departments?
(1) 240
(2) 180
(3) 150
(4) 200
(5) 220

109. What is the respective ratio between total number of female


managers from Operations and Finance departments and the total
numbers of male officers from these two departments?
Buy Printed Study Material for IBPS, SBI Bank Exams
http://www.bankexamportal.com/study-kit

Page 32

Online Coaching for IBPS, SBI Bank Exams


http://www.bankexamportal.com/elearning
(1) 4 : 5
(2) 3 : 5
(3) 1 : 2
(4) 3 : 4
(5) 2 : 3

110. Total number of male officers in Advertising and Sale


departement is what percent of the total number of officers in the
two departments?
(1) 52
(2) 54
(3) 56
(4) 58
(5) 54

111. Total number of female managers in Finance department is


what percent of the total number of male employees in sales
department?
(1) 72.5
(2) 76.25
(3) 40
(4) 82.75
(5) 88.5

112. What is the ratio between of total number of managers in


Public Relation, Finance and Sales, and Operation departments and
the total number of officers in Finance Advertising, Sales and
Procurement departments, respectively?
(1) 7 : 8
(2) 11 : 13
(3) 6 : 7
(4) 13 : 15
(5) 13 : 16
Directions (Q. Nos. 113-117) These questions consist of a question and two statements I and Ii given
below it. You have to decide whether the data providing in the statements are sufficient to answer the
question. Read both the statements and choose the appropriate option.

113. What is the speed of the train? (in km/h)


Buy Printed Study Material for IBPS, SBI Bank Exams
http://www.bankexamportal.com/study-kit

Page 33

Online Coaching for IBPS, SBI Bank Exams


http://www.bankexamportal.com/elearning
I. The car takes 2 h more than the train to cover a distance of 264 km.
II. The train moves 22 km/h faster than the car.
(1) If the data either in statement I alone or in statement Ii alone are sufficient to answer the question
(2) If the data in both the statements I and II together are not sufficient to answer the question
(3) If the data in both the statements I and II together are necessary to answer the question
(4) If the data in statement I alone are sufficient to answer the question while the data in statement II are not
sufficient to answer the question
(5) If the data in statement II alone are sufficient to answer the question while the data in statement I are not
sufficient to answer the question

114. How many students are there in the class?


I. The average weights of the class is 52 kg.
II. If two students weighing 44 kg and 52 kg leave the class and are replaced by two students weighing 64 kg
and 56 kg. The average weight of the class increases by 1.2 kg.
(1) If the data either in statement I alone or in statement II alone are sufficient to answer the question
(2) If the data in both the statements I and II together are not sufficient to answer the question
(3) If the data in both the statements I and II together are necessary to answer the question
(4) If the data in statement I alone are sufficient to answer the question while the data in statement II are not
sufficient to answer the question
(5) If the data in statement II alone are sufficient to answer the question while the data in statement I are not
sufficient to answer the question

115. In how many days can 14 men complete a piece of wouk?


I 18 women can complete that work in 24 days.
II. 28 children can complete that work in 56 days.
(1) If the data either in statement I alone or in statement II alone are sufficient to answer the question
(2) If the data in both the statements I and II together are not sufficient to answer the question
(3) If the data in both the statements I and II together are necessary to answer the question
(4) If the data in statement I alone are sufficient to answer the question while the data in statement II are not
sufficient to answer the question
(5) If the data in statement II alone are sufficient to answer the question while the data in statement I are not
sufficient to answer the question

116. What was the percentage of discount given on the marked


price of the table?
I. 15% profit was earned by selling the table for Rs. 14490. If there were not discount, percent profit would
have been 20%.
II. Cost price of table is Rs. 12600.
(1) If the data either in statement I alone or in statement II alone are sufficient to answer the question
(2) If the data in both the statement I and II together are not sufficient to answer the question
(3) If the data in both the statements I and II together are necessary to answer the question
(4) If the data in statement I alone are sufficient to answer the question while the data in statement II are not
sufficient to answer the question

Buy Printed Study Material for IBPS, SBI Bank Exams


http://www.bankexamportal.com/study-kit

Page 34

Online Coaching for IBPS, SBI Bank Exams


http://www.bankexamportal.com/elearning
(5) If the data in statement II alone are sufficient to answer the question while the data in statement I are not
sufficient to answer the question

117. What is area of the circle? (in sq cm)


I. The diameter of the circle is 90 cm less than its circumference
II. The radius of the circle is equal to the length of a rectangle whose perimeter in 77 cm.
(1) If the data either in statement I alone or in statement II alone are sufficient to answer the question
(2) If the data in both the statements I and II together are not sufficient to answer the question
(3) If the data in both the statements I and II together are necessary to answer the question
(4) If the data in statement I alone are sufficient to answer the question while the data in statement II are not
sufficient to anwer the question
(5) If the data in statement II alone are sufficient to answer the question while the data in statement I are not
sufficient to answer the question
Directions (Q. Nos. 118-121) Study the information and answer the given questions.
A bag contains four blue shirts five red shirts and six yellow shirts.s

118. Three shirts are drawn randomly. What is the probability that
exactly one of them is blue?
(1) 36/91
(2) 40/91
(3) 44/91
(4) 48/91
(5) 31/91

119. One shirt is drawn randomly. What is the probability that it is


either red or yellow?
(1) 4/15
(2) 7/15
(3) 11/15
(4) 8/15
(5) 13/15

120. Two shirts are drawn randomly. What is the probability that
both of them are blue?
(1) 3/35
(2) 1/35
(3) 2/35
(4) 5/35
(5) 6/35

Buy Printed Study Material for IBPS, SBI Bank Exams


http://www.bankexamportal.com/study-kit

Page 35

Online Coaching for IBPS, SBI Bank Exams


http://www.bankexamportal.com/elearning

121. If three shirts are drawn random 14, what is the probability
that all the three shirts are of different colour.
(1)24/91
(2)36/91
(3)12/91
(4)42/91
(5)None of these
Directions (Q. Nos. 122-126) Study the following information carefully to answer the questions. As a
part of renovation of an office flooring, painting and carpenting of a hall is to be undertaken. The hall
has length 16 m, breadth 12 m and height 5 m. Cost of flooring is Rs. 2250/m2, cost of painting is Rs.
750/m2 and cost of carpenting is Rs. 850/m2.

122. What will the total cost of flooring the hall and painting the
ceiling?
(1) Rs. 560000
(2) Rs. 576000
(3) Rs. 524000
(4) Rs. 580000
(5) Rs. 564000

123. What will be the cost of painting the four walls of the hall if it
has a door of 1.6 m 2.5 m and a window of 0.8 m 1.5 m?
(1) Rs. 190900
(2) Rs. 208000
(3) Rs. 196800
(4) Rs. 198600
(5) Rs. 206100

124. What will be the cost of flooring the hall if the contractor
offered 12% discount on the total cost?
(1) Rs. 380160
(2) Rs. 336440
(3) Rs. 328240
(4) Rs. 342320
(5) Rs. 360350

Buy Printed Study Material for IBPS, SBI Bank Exams


http://www.bankexamportal.com/study-kit

Page 36

Online Coaching for IBPS, SBI Bank Exams


http://www.bankexamportal.com/elearning

125. While painting the hall it was noticed that one wall on length
side having no door or window required plastering which will cost
Rs. 320/m2. What will be the cost of plastering and painting that
particular wall?
(1) Rs. 85600
(2) Rs. 84400
(3) Rs. 82800
(4) Rs. 86400
(5) Rs. 85200

126. What will be the difference between the cost of painting the
wall on length side and the wall on breath side neither of the walls
has a door or a window?
(1) Rs. 14000
(2) Rs. 15000
(3) Rs. 15600
(4) Rs. 16000
(5) Rs. 14500
Directions (Q. Nos. 127-132) Study the following pie-charts and the information given below them
carefully and answer the questions given.

Buy Printed Study Material for IBPS, SBI Bank Exams


http://www.bankexamportal.com/study-kit

Page 37

Online Coaching for IBPS, SBI Bank Exams


http://www.bankexamportal.com/elearning

127. The total number of which level employees in both the


organsiation together is maximum?
(1) A
(2) F
(3) C
(4) E
(5) D

128. The difference in the number of employees in the two


organisation of which level is the maximum?
(1) A
(2) D
(3) E
(4) F
(5) C

129. The number of C level employees in organisation I is


approximately equal to the number of which level employees in
organisation II?
(1) B
(2) D
(3) F
(4) C
(5) A

130. Number of B level employees in organisation I is less than that


in organisation II by what percent approximately?
(1) 60
(2) 55
(3) 59
(4) 40
(5) 45

131. What is the total number of F level employees in organisation


level I and E level employees in orgainsation II together?
(1) 7916
(2) 2652
(3) 7116

Buy Printed Study Material for IBPS, SBI Bank Exams


http://www.bankexamportal.com/study-kit

Page 38

Online Coaching for IBPS, SBI Bank Exams


http://www.bankexamportal.com/elearning
(4) 8126
(5) 4464

132. The number of A level employees in organisation II is


approximately what percentage of the number of D level employees
in organisation I?
(1) 30
(2) 62
(3) 57
(4) 48
(5) 43
Directions (Q. Nos. 133-138) Study the table and answer the given questions :

(Consider each of the given companies equally distributed book among the districutors working with it)

133. What was the average number of books published by


companies A, C and D together?
(1) 5700
(2) 5900
(3) 5500
(4) 5300
(5) 6100

134. Number of books distributed by company B is what percent of


the number of books distributed by company E?
Buy Printed Study Material for IBPS, SBI Bank Exams
http://www.bankexamportal.com/study-kit

Page 39

Online Coaching for IBPS, SBI Bank Exams


http://www.bankexamportal.com/elearning

135. Among the total number of books distributed by company D


the respective ratio of academic and non-academic books was 5: 4.
What was the number of non-academic books distributed by
company D?
(1) 1200 (2) 1800
(3) 1400
(4) 1500
(5) 1600

136. Number of books received by each distributor working with


company F was what percent more than the number of books
received by each distributor working with company E?

Buy Printed Study Material for IBPS, SBI Bank Exams


http://www.bankexamportal.com/study-kit

Page 40

Online Coaching for IBPS, SBI Bank Exams


http://www.bankexamportal.com/elearning

137. What was the respective ratio between total number of books
distributed by company A and the total number of books
distributed by company C?
(1) 53 : 88
(2) 171 : 176
(3) 53 : 92
(4) 49 : 82
(5) 57 : 92

138. What was the difference between number of books received by


each distributor working with company B and number of books
received by each distributor working with company F?
(1) 252
(2) 260
(3) 264
(4) 268
(5) 256
Directions (Q. Nos. 139-144) Study the following graph carefully and answer the questions given :

Buy Printed Study Material for IBPS, SBI Bank Exams


http://www.bankexamportal.com/study-kit

Page 41

Online Coaching for IBPS, SBI Bank Exams


http://www.bankexamportal.com/elearning

139. What is the percentage increase in the income of company Y


from 2008 to 2010?
(1) 103/2
(2) 80/3
(3) 100/3
(4) 119/3
(5) 105/4

140. The total income of all the three companies together was
minimum in which of the following years?
(1) 2011
(2) 2008
(3) 2009
(4) 2012
(5) 2007

141. In which year was the percentage increase in income of


company Z from the previous year the maximum?
(1) 2010
(2) 2008
(3) 2012
(4) 2011
(5) 2009

Buy Printed Study Material for IBPS, SBI Bank Exams


http://www.bankexamportal.com/study-kit

Page 42

Online Coaching for IBPS, SBI Bank Exams


http://www.bankexamportal.com/elearning

142. What is the approximate average income of all the three


companies together in 2011?
(1) Rs. 210 crore
(2) Rs. 240 crore
(3) Rs. 300 crore
(4) Rs. 260 crore
(5) Rs. 250 crore

143. What is the ratio between the total income of all three
companies in 2007 and 2008, respectively?
(1) 5 : 7
(2) 4 : 7
(3) 3 : 4
(4) 4 : 5
(5) 3 : 5

144. The income of company Z in 2010 is what percentage of income


of company y in 2011?

Directions (Q. Nos. 145-150) Refer to the graph and answer the given questions.

Buy Printed Study Material for IBPS, SBI Bank Exams


http://www.bankexamportal.com/study-kit

Page 43

Online Coaching for IBPS, SBI Bank Exams


http://www.bankexamportal.com/elearning

Note (i) Profit = Income Expenditure


(ii) Percent Profit = (Profit/Expenditure) = 100
(iii) Loss = Expenditure Income
(iv) Percent Loss = (Loss/Expenditure) = 100

145. What is the total profit earned by the businessman in 2005 and
2008?
(1) 50 thousand
(2) 52 thousand
(3) 55 thousand
(4) 58 thousand
(5) 60 thousand

146. What is average profit earned by the businessman in 2005,


2007 and 2009 together? (in Rs. thousand)
(1) 240
(2) 260
(3) 320
(4) 280
(5) 360

147. What is the respective ratio between total profit earned by the
businessman in 2004 and 2009 together and total loss incurred by
him in 2006 and 2008 together?
(1) 13 : 11
(2) 11 : 7
(3) 11 : 9

Buy Printed Study Material for IBPS, SBI Bank Exams


http://www.bankexamportal.com/study-kit

Page 44

Online Coaching for IBPS, SBI Bank Exams


http://www.bankexamportal.com/elearning
(4) 15 : 13
(5) 13 : 9

148. Profit earned by the businessman in 2009 is what percent more


than the profit earned by him in 2004?
(1) 162/3
(2) 221/3
(3) 122/3
(4) 142/3
(5) 181/3

149. Expenditure of the businessman increased by what percent


from 2005 to 2008?
(1) 92.5
(2) 85
(3) 95
(4) 87.5
(5) 82

150. What is the percent loss incurred by the businessman in 2006


and 2008 together?
(1) 286/13
(2) 3610/13
(3) 321/13
(4) 384/13
(5) 348/13

Buy Printed Study Material for IBPS, SBI Bank Exams


http://www.bankexamportal.com/study-kit

Page 45

Online Coaching for IBPS, SBI Bank Exams


http://www.bankexamportal.com/elearning

General Awareness, Marketing & Computer Knowledge


151. Rules for exchanging data between compuers are called
(1) Programmers (2) Hyperlinks
(3) Hyper actions
(4) Procedure
(5) Protocols

152. As per RBI guidelines, banks are no longer allowed to penalise


its customer for
(1) None-maintainers of minimum balance in cooperative accounts
(2) Non-maintained of minimum balance in current account
(3) Non-maintained of minimum balance in savings account
(4) Bouncing of cheques issued for non-availability of terms
(5) Premature withdrawal of term deposits

153. How many gigabytes are there in a prototype?


(1) 500000 (2) 100000
(3) 1000 (4) 100
(5) 001

154. Subject to compliance of security features the CBS complaint


Urban Cooperative Banks can offer the following types of internet
banking facility to their customers
(1) Non-fund based view only facility
(2) Payment of direct/indirect taxes
(3) Online-fund transfer from one account to another
(4) Online e-commerce facility
(5) Remittance of funds through NEFT/RTGS

155. Anoushka Shankar is a famous Indian


(1) Famous music director
(2) Guitarist
(3) Other than those given as options
(4) Classical singer
(5) Sitarist

Buy Printed Study Material for IBPS, SBI Bank Exams


http://www.bankexamportal.com/study-kit

Page 46

Online Coaching for IBPS, SBI Bank Exams


http://www.bankexamportal.com/elearning

156. When a customer likes a product and talks about it, this
becomes an example of
(1) Distalking
(2) Marketing
(3) Favouring
(4) Soliciting
(5) Word of mouth advertising

157. Which of the following is the autobiography of renowned


politician K. Natwar Singh?
(1) Not just an accountant
(2) One life is not enough
(3) My life with Rajiv and Sonia
(4) The substance and the Shadow
(5) Other than those given as options

158. Basically, the CTS-2010 standards issued by the Reserve Bank


of India are
(1) Cheque forms specifications
(2) Systems specifications
(3) Procedures specifications
(4) Technology specifications
(5) Equipment specification

159. Which of the following instrument is in the hands of the RBI


controls liquidity in the banking systems by regulating the money
supply in the hands of tanks that they can pump in economy?
(1) Cash revenue ratio
(2) Credit deposit ratio
(3) Other than given as options
(4) Capital adequate ratio
(5) CASA ratio

160. Which of the following banks is a public sector banks?


(1) ICICI Bank (2) IDBI Bank
(3) HDFC Bank (4) J & K Bank
(5) PNB Bank

Buy Printed Study Material for IBPS, SBI Bank Exams


http://www.bankexamportal.com/study-kit

Page 47

Online Coaching for IBPS, SBI Bank Exams


http://www.bankexamportal.com/elearning
161. The RBI has come out with a new concept of bank licensing termed as the Differentiated Bank License. A
differentiated Bank License will allow a bank to

(1) Open branches in only identified geographical areas


(2) Deal with only an identified class of customers
(3) Offer products only in select
(4) Offer services to specified institutions/customers
(5) Operate on principles of universal banking

162. The mission the Insurance Regulatory and Development


Authority is to protect the interests of the policy holders and to
(1) Ensure orderly growth of insurance industry
(2) Promote the development of financial infrastructure
(3) Specify listing and transfer of securities
(4) Regulate business of mutual funds
(5) Regulate the money and credit market

163. Which of the following schemes aims at tackling malnutrition


and health problems of children below the age of 6 years and their
memebers?
(1) Kasturba Gandhi Shishu Sangopan Yojana
(2) Kishore Vaigyanic Yojana
(3) Janani Suraksha Yojana
(4) Other than those given as options
(5) Integrated Child Development Services

164. Which of the following country(ies) have the word Dinar in


its/official currency?
1. Algeria 2. Iraq
3. Kuwait
(1) 3 and 2
(2) 1 and 2
(3) Only 1
(4) 2 and 3
(5) All of these

165. Shares of a close ended fund is setting at 20% premium over


NAV and its NAV is Rs. 10.00 market price of the

Buy Printed Study Material for IBPS, SBI Bank Exams


http://www.bankexamportal.com/study-kit

Page 48

Online Coaching for IBPS, SBI Bank Exams


http://www.bankexamportal.com/elearning
(1) Rs. 10.00
(2) Other than those given as options
(3) Rs. 11.00
(4) Rs. 12.00 (5) Rs. 8.00

166. Which button inExcel 2007 will automatically add the contents
of a group of cells?
(1) Calculate
(2) Auto add
(3) Auto sum
(4) Formula
(5) Redo

167. In which of the following cases services marketing is resorted


to?
(1) production houses
(2) only MNCs
(3) Banks and Insurance companies
(4) Public sector undertakings
(5) Industrial units

168. The SBI has recently launched a new facility called Mpassbook for its retail banking users on its State Bank Anywhere
mobile application. The facility enables users to
(1) Perform e-commerce transactions through their smartphone
(2) Bend requisition for cheque books through their smartphone
(3) Perform internet banking transactions through their smartphone
(4) Indicates view their transaction history on their smartphone
(5) remit funds under RTGS/NEFT through their smartphone

169. .............. indicates the buying process of customers.


(1) Evaluation of alternatives
(2) Separating needs and wants
(3) Purchase decision
(4) Information search
(5) Need recognition

170. The process of writing computer instructing in a programming


language is known as
Buy Printed Study Material for IBPS, SBI Bank Exams
http://www.bankexamportal.com/study-kit

Page 49

Printed Study Material for SBI PO Exam

What you will get:

100% Syllabus Covered

6 Booklets

3000+ MCQs

1100+ Pages

Guidance & Support from Our Experts

Implementation Aspects:

You will Get 6 booklets.

Study Kit will be delivered to your postal Address after payment confirmation.

After dispatching your kit we will provide you a courier tracking details.

For any help we will provide Telephonic & Email Support to the candidates.

http://www.bankexamportal.com/study-kit/sbi-po

Online Coaching for IBPS, SBI Bank Exams


http://www.bankexamportal.com/elearning
(1) Instructioning
(2) Coding
(3) Algorithming
(4) File
(5) Processing

171. The supply chain that stretches from raw materials to ............
represents a value delivery system.
(1) Final products for final buyers
(2) Warehouses for storage
(3) Production stage
(4) Suppliers
(5) Factory entrance point

172. Market Segmentation means


(1) Making selling arrangement
(2) Grouping sales team
(3) Grouping the customers as per their needs
(4) Grouping bank office staff
(5) Grouping the counter staff

173. An investor who wants to dematerialise his shares needs to


open a demat account with
(1) The securities and Exchange Board of India
(2) The concerned company
(3) Any registered share broker
(4) A recognised stock exchange
(5) A depository participant

174. Which of the following is not parts of buying decision process


of customer?
(1) Seeking discount
(2) Problem recognition
(3) Purchase design
(4) Information search
(5) Evaluation of alternatives

175. Which of the following sets of financial instruments is not


derivatives?
Buy Printed Study Material for IBPS, SBI Bank Exams
http://www.bankexamportal.com/study-kit

Page 50

Online Coaching for IBPS, SBI Bank Exams


http://www.bankexamportal.com/elearning
(1) Floors and collars
(2) Future and forwards
(3) Swaps and options
(4) Caps and credit default swaps
(5) Equities and bonds

176. Which of the following pairs of old names and new names of
countries names match of correct?
1. Constantnopie : Istambul
2. Mesopotamia : Parestine
3. Rhodesia : Zimbabwe
(1) Only 1 (2) 1 and 3
(3) All of these (4) Both 1 and 2
(5) 2 and 3

177. Which of the following is described as being too impersonal of


one way communication?
(1) Sales promotion by door-to-door campaign
(2) Personal selling
(3) Direct marketing
(4) Public relations
(5) Advertising

178. A marketing network consists of the company and its


(1) Distributors only
(2) Employees only
(3) Supporting stock holders
(4) Shareholders only
(5) Suppliers only

179. Aggressive marketing is the result of


(1) Increased job solution
(2) Outsourcing
(3) Increased marketing staff
(4) Increased competition
(5) Increased production

180. The minimum wage ceiling for becoming a subscriber for


social security schemes run by Employees Provident Fund
Organisation (EPFO) has now been raised to
Buy Printed Study Material for IBPS, SBI Bank Exams
http://www.bankexamportal.com/study-kit

Page 51

Online Coaching for IBPS, SBI Bank Exams


http://www.bankexamportal.com/elearning
(1) Rs. 1000 (2) Rs. 7500
(3) Rs. 15000 (4) Rs. 10000
(5) Rs. 1200

181. A(n) ............. analysis and executes each line of source code in
succession, without looking at the entire programme.
(1) Compiler
(2) Device driver
(3) Utility
(4) operating system
(5) Interpreter

182. Value Added Service means


(1) Old products
(2) Extra service in addition
(3) Highly valuable products
(4) Low value products
(5) Substituted products

183. Which of the following is not a part of four Ps of marketing


mix?
(1) Place (2) Price
(3) Promotion (4) Product
(5) Person

184. The decisions such as which country to enter in, how to enter
and how to introduce the products are required in
(1) Local markets (2) Global markets
(3) Social markets
(4) Government markets
(5) Service markets

185. ................ indicates working more closely with customers to add


value to the operative.
(1) Gaobasing
(2) Merging
(3) Customer partnering
(4) Re-engineering
(5) Outsourcing

Buy Printed Study Material for IBPS, SBI Bank Exams


http://www.bankexamportal.com/study-kit

Page 52

Online Coaching for IBPS, SBI Bank Exams


http://www.bankexamportal.com/elearning

186. Which of the following is a village development project under


which each member of Parliament will take the responsibility of
development physical and institutional is the infrastructure is the
identified village?
(1) Swaran Jayanti Gram Swarozgar Yojana
(2) Pradhan Mantri Gram Sadak Yojana
(3) Provision of urban
(4) National Social Assistance Programme
(5) Sansad Adarsh Gram Yojana

187. A device designed to read information encoded into a small


plastic card is
(1) Card puncher
(2) ID puncher
(3) Tape puncher
(4) Badge reader
(5) Magnetic tape

188. The Central government has recently merger of National Spot


Exchange Limited (NSEL) with which of the following?
(1) Bombay Stock Exchange
(2) Financial Technologies of India Limited (FTIL)
(3) Other than those given as options
(4) National Stock Exchange
(5) Tech Mahindra

189. Which of the following type of credit/debit cards are not


required to be swiped at the Point of Sale (PoS) terminates instead,
they have to be dipped into .............. the device and the cardholder
is required to key in a unique PIN to complete the transaction.
(1) Magnetic Stripe Base Debit Cards
(2) Magnetic Stripe Base Credit Cards
(3) Chip Based Credit/Debit Cards
(4) Cell Based Credit/Debit Cards
(5) Sim Based Credit/Debit Cards

190. The unique mobile money transfer and payment service MPesa is the trademark of
Buy Printed Study Material for IBPS, SBI Bank Exams
http://www.bankexamportal.com/study-kit

Page 53

Online Coaching for IBPS, SBI Bank Exams


http://www.bankexamportal.com/elearning
(1) HDFC Bank
(2) IDEA Cellular
(3) Vodafone
(4) ICICI Bank
(5) IDBI Bank

191. The United Nations arm UNHRC acronym stands for United
Nations
(1) Human Rights Council
(2) Human Rights Commission
(3) Human Resources Commission
(4) Other than those given as options
(5) Human Religious Committee

192. Quantitative Easing as a term which is generally used to refer


to a policy, which is adaptive?
(1) Cause money from the markets
(2) Put a ceiling on the loanable funds of banks
(3) Increase money supply in the economy
(4) Absorb excess supply in the economy
(5) Sell assets to the financial sector

193. United Nations Organisations (UNO) was set upon which of the
following dates?
(1) October 30, 1944
(2) October 31, 1945
(3) October 20, 1944
(4) October 24, 1945
(5) Other than those given as options

194. Product and Brand Management is sometimes characterised as


(1) Regional systems
(2) Segmental organisation
(3) Geographical systems
(4) Layered organisation
(5) Hub and spoke system

195. The ALU and control unit of most of the microcomputer are
combined and manufactured in a single silicon chip. What it is
called?
Buy Printed Study Material for IBPS, SBI Bank Exams
http://www.bankexamportal.com/study-kit

Page 54

Online Coaching for IBPS, SBI Bank Exams


http://www.bankexamportal.com/elearning
(1) ALU
(2) Touchpad
(3) Control Unit
(4) Men chip
(5) Microprocessor

196. Jeffrey Fournie, an American was freed unexpectedly recently


by the government of
(1) North Korea
(2) Other than those given as options
(3) Syria
(4) South Korea
(5) Taiwan

197. National Chemical Laboratory is located at


(1) New Delhi
(2) Chandigarh
(3) Hyderabad
(4) Pune
(5) Other than those given as options

198. Which of the following is used to write webpages?


(1) HTML
(2) Telnet
(3) HTTP
(4) URL
(5) FTP

199. Which of the following is not a valid version of MS-Office?


(1) Office XP (2) Office 2003
(3) Office 2004
(4) Office Vista
(5) Office 2003

200. URL stands for


(1) Uniform Retail Location
(2) Universal Resource List
(3) Uniform Research Locator
(4) Universal Research List
(5) Uniform Resource Locator

Buy Printed Study Material for IBPS, SBI Bank Exams


http://www.bankexamportal.com/study-kit

Page 55

Online Coaching for IBPS, SBI Bank Exams


http://www.bankexamportal.com/elearning

SBI PO Exam Paper - 2013


Data Analysis and Interpretation
Directions: Study the following graph carefully to answer the questions that follow:
Cost of three different fruits (in rupees per kg. in five different cities)

1. In which city is the difference between the cost of one kg of apple and cost of one kg of guava second
lowest?
(1) Jalandhar (2) Delhi
(3) Chandigarh (4) Hoshiarpur
(5) Ropar
2. Cost of one kg of guava in Jalandhar is approximately what percent of the cost of two kgs of grapes
in Chandigarh?
(1) 66 (2) 24
(3) 28 (4) 34
(5) 58
3. What total amount will Ram pay to the shopkeeper for purchasing 3 kgs of apples and 2 kgs of
guavas in Delhi?

Buy Printed Study Material for IBPS, SBI Bank Exams


http://www.bankexamportal.com/study-kit

Page 56

Online Coaching for IBPS, SBI Bank Exams


http://www.bankexamportal.com/elearning
(1) Rs. 530/- (2) Rs. 450/(3) Rs. 570/- (4) Rs. 620/(5) Rs. 490/4. Ravinder had to purchase 45 kgs of graphs from Hoshiarpur. Shopkeeper gave him discount of 4%
per kg. What amount did he pay to the shopkeeper after the discount?
(1) Rs. 8208/- (2) Rs. 8104/(3) Rs. 8340/- (4) Rs. 8550/(5) Rs. 8410/5. What is the respective ratio between the cost of one kg of apples from Ropar and the cost of one kg
of grapes from Chandigarh?
(1) 3 : 2 (2) 2 : 3
(3) 22 : 32 (4) 42 : 92
(5) 92 : 42
Directions: Study the radar graph carefully and answer the questions that follow:
Number of students (in thousands) in two different universities in six different years
Number of Students

Buy Printed Study Material for IBPS, SBI Bank Exams


http://www.bankexamportal.com/study-kit

Page 57

Online Coaching for IBPS, SBI Bank Exams


http://www.bankexamportal.com/elearning
6. What was the difference between the number of students in university-1 in the year 2010 and the
number of students in university-2 in the year 2012?
(1) Zero (2) 5,000
(3) 15,000 (4) 1,0000
(5) 1,000
7. What is the sum of number of students in university-1 in the year 2007 and the number of students
in university-2 in the year 2011 together?
(1) 50,000 (2) 55000
(3) 45000 (4) 57000
(5) 40000
8. If 25% of the students in university-2 in the year 2010 were females, what was the number of male
students in the university-2 in the same year?
(1) 11250 (2) 12350
(3) 12500 (4) 11500
(5) 11750
9. What was the percent increase in the number of students in University-1 in the year 2011 as
compared to the previous year?
(1) 135 (2) 15
(3) 115 (4) 25
(5) 35
10. In which year was the difference between the number of students in university-1 and the number
of students in univeisity-2 highest?
(1) 2008 (2) 2009
(3) 2010 (4) 2011
(5) 2012
Directions: Study the graph carefully to answer the questions that follow
Number of players participating in three different games in five different schools

Buy Printed Study Material for IBPS, SBI Bank Exams


http://www.bankexamportal.com/study-kit

Page 58

Online Coaching for IBPS, SBI Bank Exams


http://www.bankexamportal.com/elearning

11. What is the total number of players participating in hockey from all the five schools together?
(1) 324 (2) 288
(3) 342 (4) 284
(5) 248
12. What is the respective ratio between number of players participating in basketball from school-1
and the number of players participating in Kho-Kho from school-3?
(1) 5 : 7 (2) 7 : 9 (3) 7 : 8 (4) 9 : 7
(5) 5 : 8
13. In which school is the number of players participating in hockey and basketball together second
highest?
(1) School 1 (2) School 2
(3) School 3 (4) School 4
(5) School 5
14. Number of players participating in Kho-Kho from school-4 is what percent of number of players
participating in hockey from school-2?
(1) 42 (2) 48 (3) 36 (4) 40
(5) 60
15. 25% of the number of the players participating in hockey from School-5 are females. What is the
number of the hockey players who are males in school-5?
(1) 15 (2) 18 (3) 30 (4) 21
(5) 27
Directions: Study the following bar-graph carefully and answer the following questions.

Buy Printed Study Material for IBPS, SBI Bank Exams


http://www.bankexamportal.com/study-kit

Page 59

Online Coaching for IBPS, SBI Bank Exams


http://www.bankexamportal.com/elearning
Earnings (in rupees) of the three different persons on four different days

16. What is Gitas average earnings over all the days together?
(1) Rs. 285 (2) Rs. 290
(3) Rs. 320 (4) Rs. 310
(5) Rs. 315
17. What is the total amount earned by Rahul and Naveen together on Tuesday and Thursday
together?
(1) Rs. 1040/- (2) Rs. 1020/(3) Rs. 980/- (4) Rs. 940/(5) Rs. 860/18. Gita donated her earnings of Wednesday to Naveen. What was Naveens total earnings on
Wednesday after Gitas donation?
(1) Rs. 520/- (2) Rs. 550/(3) Rs. 540/- (4) Rs. 560/(5) Rs. 620/19. What is the difference between Rahuls earnings on Monday and Gitas earnings on Tuesday?
(1) Rs. 40/- (2) Rs. 20/(3) Rs. 50/- (4) Rs. 30/(5) Rs. 10/20. What is the respective ratio between Naveens earnings on Monday, Wednesday and Thursday?

Buy Printed Study Material for IBPS, SBI Bank Exams


http://www.bankexamportal.com/study-kit

Page 60

Online Coaching for IBPS, SBI Bank Exams


http://www.bankexamportal.com/elearning
(1) 7 : 3 : 5 (2) 8 : 6 : 5
(3) 8 : 7 : 4 (4) 9: 5 : 4
(5) 9 : 3 : 4
Directions: Study the following pie-chart and answer the following questions.
Percentage-wise distribution of employees in six different professions
Total number of employees = 26800

21. What is the difference between the total number of employees in teaching and medical profession
together and number of employees in management profession?
(1) 6770 (2) 7700
(3) 6700 (4) 7770
(5) 7670
22. In management profession three-fourth of the number of employees are female. What is the
number of male employees in management profession?
(1) 1239 (2) 1143
(3) 1156 (4) 1289
(5) 1139
23. 25% of employees from film production profession went on a strike. What is the number of
employees from film production who have not participated in the strike?
(1) 3271 (2) 3819
(3) 3948 (4) 1273
(5) 1246

Buy Printed Study Material for IBPS, SBI Bank Exams


http://www.bankexamportal.com/study-kit

Page 61

Online Coaching for IBPS, SBI Bank Exams


http://www.bankexamportal.com/elearning
24. What is the total number of employees in engineering profession and industries together?
(1) 5698 (2) 5884
(3) 5687 (4) 5896
(5) 5487
25. In teaching profession if three-fifth of the teachers are not permanent, what is the number of
permanent, what is the number of permanent teacher in the teaching professions?
(1) 1608 (2) 1640
(3) 1764 (4) 1704
(5) 1686
Direction: Study the table carefully to answer the questions that follow:
Monthly Bill (in rupees) of landline phone, electricity, laundry and mobile phone paid by three
different people in five different months

26. What is the total of bill paid by Dev in the month of June for all the four commodities?
(1) Rs. 608 (2) Rs. 763
(3) Rs. 731 (4) Rs. 683
(5) Rs. 674
27. What is the average electricity bill paid by Manu over all the five months together?
(1) Rs. 183 (2) Rs. 149
(3) Rs. 159 (4) Rs. 178
(5) Rs. 164
28. What is the difference between the mobile phone bill paid by Ravi in the month of May and the
laundry bill paid by Dev in the month of March?
(1) Rs. 180 (2) Rs. 176
(3) Rs. 190 (4) Rs. 167
(5) Rs. 196
29. In which months respectively did Manu pay the second highest mobile phone bill and the lowest
electricity bill?

Buy Printed Study Material for IBPS, SBI Bank Exams


http://www.bankexamportal.com/study-kit

Page 62

Online Coaching for IBPS, SBI Bank Exams


http://www.bankexamportal.com/elearning
(1) April and June (2) April and May
(3) March and June (4) March and May
(5) July and May
30. What is the respective ratio between the electricity bill paid by Manu in the month of April and
the mobile phone bill paid by Ravi in the month of June?
(1) 27 : 49 (2) 27 : 65
(3) 34 : 49 (4) 135 : 184
(5) 13 : 24
Directions: Study the following table carefully and answer the questions that follow:
Chart showing schedule of train from Dadar to Bhuj and number of passengers boarding from each
station

31. What is the distance travelled by the train from Surat to Nadiad Jn. ?
(1) 176 km (2) 188 km
(3) 183 km (4) 193 km
(5) 159 km
32. How much time does the train take to reach Ahmedabad after departing from Anand Jn.
(including the halt time)?
(1) 1 hr. 59 min. (2) 1 hr. 17 min.
(3) 1 hr. 47 min. (4) 1 hr. 45 min.
(5) 1 hr. 15 min.
33. What is the respective ratio between the number of passengers boarding from Vasai Road and
from Ahmadabad in the train?
(1) 21 : 17 (2) 13 : 9
(3) 21 : 19 (4) 15 : 13
(5) 13 : 15

Buy Printed Study Material for IBPS, SBI Bank Exams


http://www.bankexamportal.com/study-kit

Page 63

Online Coaching for IBPS, SBI Bank Exams


http://www.bankexamportal.com/elearning
34. If halt time (stopping time) of the train Vadodara is decreased by 2 minutes and increased by 23
minutes at Ahmedabad. At what time will the train reach Bhuj?
(1) 6.10 am (2) 6.01 pm
(3) 6.05 pm (4) 6.50 pm
(5) 6.07 pm
35. Distance between which two station is second lowest?
(1) Nadiad Jn. To Ahmedabad
(2) Anand Jn. To Nadiad Jn.
(3) Dadar to Vasai Road
(4) Anand Jn. To Vadodara
(5) Vasai Road to Surat
Directions: Study the table carefully to answer the question that follow.
Maximum and minimum temperature (in degree Celsius) recorded on 1st day of each month of five
different cities

36. What is the difference between the maximum temperature of Ontario on 1st November and the
minimum temperature of Bhuj on 1st January?
(1) 3C (2) 18C
(3) 15C (4) 9C
(5) 11C
37. In which month respectively the maximum temperature of Kabul is second highest and minimum
temperature of Sydney is highest?
(1) 1st October and 1st January
(2) 1st October and 1st November
(3) 1st December and 1st January
(4) 1st September and 1st January
(5) 1st December and 1st September
38. In which month (on 1st day) is the difference between maximum temperature and minimum
temperature of Bhuj second highest?

Buy Printed Study Material for IBPS, SBI Bank Exams


http://www.bankexamportal.com/study-kit

Page 64

Online Coaching for IBPS, SBI Bank Exams


http://www.bankexamportal.com/elearning
(1) 1st September
(2) 1st October
(3) 1st November
(4) 1st December
(5) 1st January
39. What is the average maximum temperature of Beijing over all the months together?
(1) 8.4C
(2) 9.6C
(3) 7.6C
(4) 9.2C
(5) 8.6C
40. What is the respective ratio between the minimum temperature of Beijing on 1st September and
the maximum temperature of Ontario on 1st October?
(1) 3 : 4 (2) 3 : 5
(3) 4 : 5 (4) 1 : 5
(5) 1 : 4
Directions: Study pie-chart and table carefully to answer the question that follow:
Pie-chart showing percentage-wise distribution of cars in four different states
Total cars = 700
Distribution of Cars

Buy Printed Study Material for IBPS, SBI Bank Exams


http://www.bankexamportal.com/study-kit

Page 65

Online Coaching for IBPS, SBI Bank Exams


http://www.bankexamportal.com/elearning

41. What is the difference between the number of diesel engine cars in state-2 and the number of
petrol engine cars in state-4?
(1) 159 (2) 21
(3) 28 (4) 34
(5) 161
42. Number of petrol engine cars in state-3 is what percent more than the number of diesel engine cars
in state-1?
(1) 100
(2) 200
(3) 300
(4) 125
(5) 225
43. If 25% of diesel engine cars in state-3 are AC and remaining cars are non-AC, what is the number
of diesel engine cars in state-3 which are non-AC?

Buy Printed Study Material for IBPS, SBI Bank Exams


http://www.bankexamportal.com/study-kit

Page 66

Online Coaching for IBPS, SBI Bank Exams


http://www.bankexamportal.com/elearning
(1) 75
(2) 45
(3) 95
(4) 105
(5) 35
44. What is the difference between the total number of cars in state-3 and the number of petrol engine
cars in state-2?
(1) 96 (2) 106
(3) 112
(4) 102
(5) 98
45. What is the average number of petrol engine cars in all the states together?
(1) 86.75 (2) 89.25
(3) 89.75 (4) 86.25
(5) 88.75
46. A bag contains 7 blue balls and 5 yellow balls. If two balls are selected at random, what is the
probability that none is yellow?
(1) (2)
(3) (4)
(5)
47. A die is thrown twice. What is the probability of getting a sum of 7 from both the throws?
(1) (2)
(3) (4)
(5)
Directions: Study the information carefully to answer these questions.
In a team there are 240 members (males and females). Two-third of them are males. Fifteen percent of males
are graduates. Remaining males are non-graduates. Three-fourth of the females are graduates. Remaining
females are non-graduates.
48. What is the difference between the number of females who are non-graduates and the number of
males who are graduates?
(1) 2
(2) 24
(3) 4
(4) 116
(5) 36

Buy Printed Study Material for IBPS, SBI Bank Exams


http://www.bankexamportal.com/study-kit

Page 67

Online Coaching for IBPS, SBI Bank Exams


http://www.bankexamportal.com/elearning
49. What is the sum of number of females who are graduates and the number of males who are nongraduates?
(1) 184
(2) 96
(3) 156
(4) 84
(5) 196
50. What is the ratio between the total number of males and the number of females who are nongraduates?
(1) 6 : 1
(2) 8 : 1
(3) 8 : 3
(4) 5 : 2
(5) 7 : 2

Buy Printed Study Material for IBPS, SBI Bank Exams


http://www.bankexamportal.com/study-kit

Page 68

Online Coaching for IBPS, SBI Bank Exams


http://www.bankexamportal.com/elearning

Reasoning Ability
Directions (1-5): Read each statement carefully and answer the following questions:
1. Which of the following expressions will be true if the expression R > O = A > S < T as definitely
true?
(1) O > T (2) S < R
(3) T > A (4) S = O
(5) T < R
2. Which of the following symbols should replace the questions mark (?) in the given expression in
order to make the expression P > A as well as T L definitely true?
P>L?AN=T
(1) (2) <
(3) < (4)
(5) Either or <
3. Which of the following symbols should be placed in the blank spaces respectively (in the same order
from left to right) in order to complete the given such a manner that makes the expressions B>N as
well as D L definitely true?
B_____L_____O____N_____D
(1) =, =, ,
(2) >, , =, > (3) >, <, =,
(4) >, =, =, (5) >, =, , >
4. Which of the following should be placed in the blank spaces respectively (in the same order from
left to right) in order to complete the given expression in such a manner that makes the expression A
< P definitely false?
____ < ___ < ___ > ___
(1) L, N, P, A (2) L, A, P, N
(3) A, L, P, N (4) N, A, P, L
(5) P, N, A, L
5. Which of the following symbols should be placed in the blank spaces respectively (in the same order
from left to right) in order to complete the given expression in such a manner that makes the
expression F > N and U > D definitely true?
F__O__U__N__D

Buy Printed Study Material for IBPS, SBI Bank Exams


http://www.bankexamportal.com/study-kit

Page 69

Online Coaching for IBPS, SBI Bank Exams


http://www.bankexamportal.com/elearning
(1) <, <, >, = (2) <, =, =, >
(3) <, =, =, < (4) , =, =, >
(5) >, >, =, <
Directions (6-10): Study the following and answer the following questions:
A, B, C, D, E, G and I are seven friends who study in three different standards namely 5th, 6th and 7th such
that not less than two friends study in the same standard. Each friend also has a different favorite subject
namely History, Civics, English, Marathi, Hindi, Maths and Economics but not necessarily in the same
order.
A likes Math and studies in the 5th standard with only one other friend who likes Marathi. I studies with two
other friends. Both the friends who study with I like languages (Here languages include only Hindi, Marathi
and English). D Studies in the 6th standard with only one person and does not like civics. E studies with
only one friend. The one who likes history does not study in 5th or 6th standard. E does not like languages.
C does not like English, Hindi or Civics.
6. Which combination represents Es favourite subject and the standard in which he studies?
(1) Civics and 7th
(2) Economics and 5th
(3) Civics and 6th
(4) History and 7th
(5) Economics and 7th
7. Which of the following is Is favourite subject?
(1) History
(2) Civics
(3) Marathi
(4) Either English or Marathi
(5) Either English or Hindi
8. Which of the following is definitely correct?
(1) G (2) C
(3) E (4) D
(5) Either D or B
9. Which of the following is definitely correct?
(1) I and Hindi (2) G and English
(3) C and Marathi (4) B and Hindi
(5) E and Economics
10. Which of the following subjects does G like?

Buy Printed Study Material for IBPS, SBI Bank Exams


http://www.bankexamportal.com/study-kit

Page 70

Online Coaching for IBPS, SBI Bank Exams


http://www.bankexamportal.com/elearning
(1) Either Maths or Marathi
(2) Either Hindi or English
(3) Either Hindi or Civics
(4) Either Hindi or Marathi
(5) Either Civics or Economics
Direction: Study the information and answer the following questions:
In a certain code language- economics is not money is written as ka la ho ga demand and supply
economics is written as, mo ta pa ka money makes only part is written as, zi la ne ki demand
makes supply economics is written as, zi mo ka ta
11. What is the code for money in the given code language?
(1) ga (2) mo
(3) pa (4) ta
(5) la
12. What is the code for supply in the given code language?
(1) only ta (2) only mo
(3) either pa or mo (4) only pa
(5) either mo or ta
13. What is the code for demand only more in the given code language?
(1) xi ne mo (2) mo zi ne
(3) ki ne mo (4) mo zi ki
(5) xi ka ta
14. What may be the possible code for work and money in the given code language?
(1) pa ga la (2) pa la tu
(3) mo la pa (4) tu la ga
(5) pa la ne
15. What is the code for makes in the given code language?
(1) mo (2) pa
(3) ne (4) zi
(5) ho
Directions: Study the given information and answer the following questions:
When a word and number arrangement machine is given an input line of words and numbers, it arranges
them following a particular rule. The following is an illustration of input and rearrangement: (All the
numbers are two digit numbers)

Buy Printed Study Material for IBPS, SBI Bank Exams


http://www.bankexamportal.com/study-kit

Page 71

Online Coaching for IBPS, SBI Bank Exams


http://www.bankexamportal.com/elearning
Input: 40 made butter 23 37 cookies salt extra 52 86 92 fell now 19
Step I: butter 19 40 made 23 37 cookies salt extra 52 86 92 fell now
Step II: cookies 23 butter 19 40 make 37 salt extra 52 86 92 fell now
Step III: extra 37 cookies 23 butter 19 40 made salt 52 86 92 fell now
Step IV: fell 40 extra 37 cookies 23 butter 19 made salt 52 86 92 now
Step V: made 52 fell 40 extra 37 cookies 23 butter 19 salt 86 92 now
Step VI: now 86 made 52 fell 40 extra 37 cookies 23 butter 19 salt 92
Step VII: salt 92 now 86 made 52 fell 40 extra 37 cookies 23 butter 19
Step VII is the last step of the above arrangement as the intended arrangement is obtained.
As per the rules followed in the given steps, find out he appropriate steps for the given input.
Input: 32 proud girl beautiful 48 55 97 rich family 61 72 17 nice life
16. How many steps will be required to complete the given input?
(1) Five (2) Six
(3) Seven (4) Eight
(5) Nice
17. Which of the following is the third element from the left end of step VI?
(1) beautiful (2) life
(3) 61 (4) Nice
(5) 17
18. Which of the following is step III of the given input?
(1) proud 72 girl 48 family 32 beautiful 17 55 97 rich 61 nice life
(2) life 55 girl 48 family 32 beautiful 17 proud 97 rich 61 72 nice
(3) girl 48 family 32 beautiful 17 proud girl 48 55 97 rich 61 72 nice life
(4) family 32 beautiful 17 proud girl 48 55 97 rich 61 72 nice life
(5) girl 48 life 55 family 32 beautiful 17 proud 97 rich 61 72 nice
19. What is the position of nice from the left end in the final step?

Buy Printed Study Material for IBPS, SBI Bank Exams


http://www.bankexamportal.com/study-kit

Page 72

Online Coaching for IBPS, SBI Bank Exams


http://www.bankexamportal.com/elearning
(1) Fifth (2) Sixth
(3) Seventh (4) Eight
(5) Ninth
20. Which element is third to the right of family in Step V?
(1) beautiful (2) 17
(3) Proud (4) 97
(5) 32
Directions: Read the information carefully and answer the following questions:
If A + B means A is the father of B
If A B means A is the sister of B
If A $ B means A is the wife of B
If A % B means A is the mother of B
If A B means A is the son of B
21. What should come in place of the question mark, to establish that J is the brother of T in the
expression?
JP%H? T%L
(1) (2)
(3) $ (4) Either or
(5) Either + or
22. Which among the given expression indicate the M is the daughter of D?
(1) L % R $ D + T M
(2) L + R $ D + M T
(3) L % R % D + T M
(4) D + L $ R M + T
(5) L $ D R % M T
23. Which among the following options is true if the expression I + T % J L K is definitely true?
(1) L is the daughter of T
(2) K is the son-in-law of I
(3) I is the grandmother of L
(4) T is the father of J
(5) J is the brother of L
24. Which among the following expression of true if Y is the son of X is definitely false?
(1) W % L T Y X
(2) W + L T Y X
(3) X + L T Y W

Buy Printed Study Material for IBPS, SBI Bank Exams


http://www.bankexamportal.com/study-kit

Page 73

Online Coaching for IBPS, SBI Bank Exams


http://www.bankexamportal.com/elearning
(4) W $ X + L + Y + T
(5) W % X + T Y L
25. What should come in place of the question mark, to establish that T is the sister-in-law of Q in the
expression?
R%TP?Q+V
(1) (2) %
(3) (4) $
(5) Either $ or
Directions: Study the following information and answer the questions given below:
Eight people- E, F, G, H, J, K, L and M are sitting around a circular table facing the centre. Each of them is
of a different profession Charted Accountant, Columnist, Doctor, Engineer, Financial Analyst, Lawyer,
Professor and Scientist but not necessarily in the same order. F is sitting second to the left of K. the Scientist
is an immediate neighbour of K. there are only three people between the Scientist and E only one person sits
between the Engineer and E. The Columnist is to the immediate right of the Engineer. M is second to the
right of K. H is the Scientist. G and J are immediate neighbours of each other. Neither G nor J is an
Engineer. The Financial Analyst is to the immediate left of F. The Lawyer is second to the right of the
Columnist. The Professor is an immediate neighbor of the Engineer. G is second to the right of the Chartered
Account.
26. Who is sitting second to the right of E?
(1) The Lawyer (2) G
(3) The Engineer (4) F
(5) K
27. Who amongst the following is the Professor?
(1) F (2) L (3) M (4) K
(5) J
28. Four of the following five are alike in a certain way based on the given arrangement and hence
form a group. Which of the following does not belong to that group?
(1) Chartered Account H
(2) M - Doctor (3) J Engineer
(4) Financial Analyst L
(5) Lawyer K
29. What is the position of L with respect to the Scientist?
(1) Third to the left
(2) Second to the right
(3) Second to the left

Buy Printed Study Material for IBPS, SBI Bank Exams


http://www.bankexamportal.com/study-kit

Page 74

Online Coaching for IBPS, SBI Bank Exams


http://www.bankexamportal.com/elearning
(4) Third to the right
(5) Immediate right
30. Which of the following statements is true according to the given arrangement?
(1) The Lawyer is second to the left of the doctor
(2) E is an immediate neighbor of the Financial Analyst
(3) H sits exactly between F and the Financial Analyst
(4) Only four people sit between the Columnist and F
(5) All of the given statement are true
Directions: In each of the questions below, two/three/ statements are given followed by conclusions/group
of conclusions numbered I and II. You have to assume all the statements to be true even if they seem to be at
variance from the commonly known facts and then decide which of the given two conclusions logically
follows from the information given in the statements.
Give answer (1) if only conclusion I follows
Give answer (2) if only conclusion II follows
Give answer (3) if either I or II follows
Give answer (4) if neither I nor II follow
Give answer (5) if both I and II follow
31-32. Statements:
Some squares are circles.
No circle is a triangle.
No line is a square.
31. Conclusions:
I. All squares can never be triangles.
II. Some lines are circles.
32. Conclusions:
I. No triangle is a square.
II. No line is a circle.
33-34. Statements:
All songs are poems.
All poems are rhymes.
No rhyme is a paragraph.
33. Conclusions:

Buy Printed Study Material for IBPS, SBI Bank Exams


http://www.bankexamportal.com/study-kit

Page 75

Online Coaching for IBPS, SBI Bank Exams


http://www.bankexamportal.com/elearning
I. No song is a paragraph.
II. No poem is a paragraph.
34. Conclusions:
I. All rhymes are poems.
II. All songs are rhymes.
35. Statements:
Some dews are drops.
All drops are stones.
Conclusions:
I. Atleast some dews are stones.
II. Atleast some stones are drops.
Directions: Each of the questions below consists of a question and two statements numbered I and II given
below it. You have to decide whether the data given in the statements are sufficient to answer the questions.
Read both the statements and
Give answer (1) if the data in statement I alone are sufficient to answer the question, while the data in
statement II alone are not sufficient to answer the question.
Give answer (2) if data in statement II alone are sufficient to answer the question, while the data in statement
I alone are not sufficient to answer the question.
Give answer (3) if the data either in statement I alone or statement II alone are sufficient to answer the
questions.
Give answer (4) if the data neither in statement I nor II together are sufficient to answer the questions.
Give answer (5) if the data in both statements I and II together are necessary to answer the questions.
36. Seventeen people are standing in a straight line facing south. What is Bhavnas positon from the
left end of the line?
(1) Sandeep is standing second to the left of Sheetal. Only five people stand between Sheetal and the one
who is standing at the extreme right end of the live. Four people stand between Sandeep and Bhavna.
(2) Anita is standing fourth to the left of Sheetal. Less than three people are standing between Bhavna and
Anita.
37. Five letters A, E, G, N and R are arranged to right according to certain conditions. Which letter
is placed third?
I. G is placed second to the right of A. E is to the immediate right of G. there are only two letters between R
and G.
II. N is exactly between A and G. Neither A nor G is at the extreme end of the arrangement.
38. Six people S, T, U, V, W and X are sitting around a circular table facing the centre. What is Ts
position with respect to X?

Buy Printed Study Material for IBPS, SBI Bank Exams


http://www.bankexamportal.com/study-kit

Page 76

Online Coaching for IBPS, SBI Bank Exams


http://www.bankexamportal.com/elearning
I. Only two people sit between U and W. X is second to the left of W. V and T are immediate neighbours of
each other.
II. T is to the immediate right of V. There are only two people between T and S. X is an immediate
neighbour of S but not of V:
Directions: Read the following information carefully and answer the given question:
The convenience of online shopping is what I like best about it. Where else can you shop even at midnight
wearing your night suit? You do not have to wait in a line or wait till the shop assistant is ready to help you
with your purchases. It is a much better experience as compared to going to a retail store. A consumers
view.
39. Which of the following can be a strong argument in favour of retail store owners?
(1) Online shopping portals offer a great deal of discounts which retail stores offer only during the sale
season
(2) One can compare a variety of products online which cannot be done at retail stores
(3) Many online shopping portals offer the cash on delivery feature which is for those who are sveptical
about online payments
(4) Many consumers prefer shopping at retail stores which are nearer to their houses
(5) In online shopping the customer may be deceived as he cannot touch the product he is paying for.
40. Which of the following can be inferred from the given information? (An= inference is something
that is not directly stated but can be inferred from the given information)
(1) One can shop online only at night
(2) Those who are not comfortable using computers can never enjoy the experience of online shopping
(3) All retail stores provide shopping assistants to each and every customer
(4) The consumer whose view is presented has shopped at retail stores as well as online
(5) The customer whose view is presented does not have any retail stores in her vicinity
41. Read the following information carefully and answer the given question:
Many manufacturing companies are now shifting base to the rural areas of the country as there is a scarcity
of space in urban areas. Analysts say that this shift will not have a huge impact on the prices of the products
manufactured by these companies as only about 30% consumers live in urban areas. Which of the following
maybe a consequence of the given information?
(1) The prices of such products will decrease drastically in the urban areas
(2) People living in urban areas will not be allowed to work in such manufacturing companies
(3) These manufacturing companies had set-ups in the urban areas before shifting base
(4) Those who had already migrated to the urban areas will not shift back to rural areas
(5) The number of people migrating from rural to urban areas in search of jobs may reduce
42. Read the following information carefully and answer the given question:
Pets are not allowed in the park premises A notice put up at the park entrance by the authority
that is responsible for maintenance of the park.

Buy Printed Study Material for IBPS, SBI Bank Exams


http://www.bankexamportal.com/study-kit

Page 77

Online Coaching for IBPS, SBI Bank Exams


http://www.bankexamportal.com/elearning
Which of the following can be an assumption according to the given information? (An assumption is
something that is supposed or taken for granted)
(1) At least some people who visit the park have pets.
(2) This is the only park which does not allows pets
(3) People who ignored this notice were fined
(4) There are more than one entrances to the park
(5) Many people have now stopped visiting the park
Directions: Read the following information carefully and answer the given questions:
Despite repeated announcements that mobile phones were not allowed in the examination hall, three
students were caught with their mobile phones.
(A) Mobile phones nowadays have a lot of features and it is easy to cheat with their help
(B) The invigilator must immediately confiscate the mobile phones and ask the students to leave the exam
hall immediately.
(C) Mobile phones are very expensive and leaving them in bags outside the exam hall is not safe.
(D) There have been incidents where students who left the exam hall early stole the mobile phones kept in
the bags of the students who were writing the exam
(E) The school authorities must ask the students to leave their phones in the custody of the invigilator before
the exam in order to avoid thefts of mobile phones
(F) None of the other students were carrying their phones in the exam hall.
43. Which of the following among (A), (B), (C) and (D) may be a strong argument in favour of, the
three students who were caught with the mobile phone?
(1) Only (A) (2) Both (A) and (B)
(3) Both (C) and (D) (4) Only (C)
(5) Both (B and (D)
44. Which of the following among (A), (B), (E) and (F) may be the reason behind the school making
such announcements before the exam?
(1) Only (B) (2) Both (B) and (E)
(3) Only (F) (4) Only (A)
(5) Both (E) and (A)
45. Which of the following among (A), (B), (D) and (F) can be an immediate course of action for the
invigilator?
(1) Only (B) (2) Both (A) and (D)
(3) Only (A) (4) Both (D) and (F)
(5) Only (F)
Directions: In each of the questions given below which one of the five answer on the right should come
after the problem figure on the left, it the sequence were continued?

Buy Printed Study Material for IBPS, SBI Bank Exams


http://www.bankexamportal.com/study-kit

Page 78

Online Crash Course for SBI PO Exam


What candidate will get:

All the relevant and required materials of subjects mentioned in the SBI PO NEW Syllabus (Worth
Rs.699):

100% SBI PO Exam Syllabus Covered with MCQs.

General Awareness Magazine for IBPS, SBI, & All Bank Exams

Descriptive test

10 Practice Tests Series with Ans (Worth Rs.499)

Current Affairs Subscription (1 Year Subscription - 12 Pdf Copy)

Special Current Affairs for SBI PO Exam (6 Months - Pdf Copy)

Previous Year Solved Papers

http://www.bankexamportal.com/online-coaching/sbi-po-crash-course

Online Coaching for IBPS, SBI Bank Exams


http://www.bankexamportal.com/elearning

Buy Printed Study Material for IBPS, SBI Bank Exams


http://www.bankexamportal.com/study-kit

Page 79

Online Coaching for IBPS, SBI Bank Exams


http://www.bankexamportal.com/elearning

English Language
Directions: Read the following passage carefully and answer the questions given below it. Certain words
have been printed in bold to help you locate them while answering some of the questions.
Financial Inclusion (FI) is an emerging priority for banks that have nowhere else to go to achieve business
growth. The viability of FI business is under question, because while banks and their delivery partners continue
to make investments, they havent seen commensurate returns. In markets like India, most programs are
focused on customer on-boarding, an expensive process which people often find difficult to afford, involving
issuance of smart cards to the customers. However, large scale customer acquisition hasnt translated into
large scale customer acquisition hasnt translated into large scale business, with many accounts lying dormant
and therefore yielding no return on the banks investment. For the same reason, Business Correspondent
Agents who constitute the primary channel for financial inclusion are unable to pursue their activity as a fulltime job. On major reason for this state of events is that the customer ob-boarding process is often delayed
after the submission of documents (required to validate the details of the concerned applicant) by the applicant
and might take as long as two weeks. By this time the initial enthusiasm of applicants fade away. Moreover,
the delivery partners dont have the knowledge and skill to propose anything other than the most basic financial
products to the customer and hence do not serve their banks goal of expanding the offering in unbanked
markets.
Contrary to popular perception, the inclusion segment is not a singular impoverished, undifferentiated mass
and it is important to navigate its diversity to identify the right target customers for various programs. Rural
markets do have their share of rich people who do not use banking services simply because they are
inconvenient to access or have low perceived value. At the same time, urban markets, despite a high branch
density, have multitude of low wage earners outside the financial net. Moreover, the branch timing of banks
rarely coincide with the off-work hours of the labour class.
Creating affordability is crucial in tapping the unbanked market. No doubt pricing is a tool, but banks also
need to convince customers that they can derive big value even from small amounts. One way of doing this is
to show the target audience that a bank account is actually a lifestyle enabler, a convenient and safe means to
send money to family or make a variety of purchases. Once banks succeed in hooking customers with this
value proposition they must sustain their interest by introducing a simple and intuitive use application,
ubiquitous access over mobile and other touch points, and adopting a banking mechanism which is not only
secure but also reassuring to the customer. Technology is the most important element of financial inclusion
strategy and an enabler of all others. The choice of technology is therefore a crucial decision, which could
make or mar the agenda. Of the various selection criteria, cost is perhaps the most important. This certainly
does not mean buying the cheapest package, but rather choosing that solution which by scaling transactions
to huge volumes reduces per unit operating cost. An optimal mix of these strategies cost. An optimal mix of
these strategies would not doubt offer an innovative means of expansion in the unbanked market.
1. Which of the following facts is true as per the passage?
(1) People from rural areas have high perceived value of banking services
(2) Cost is not a valid criterion for technological package selection for financial inclusion initiatives
(3) The inclusion segment is a singular impoverished, undifferentiated mass
(4) The branch timings of banks generally do not coincide with the off-work hours of the labour class in
urban markets
(5) All the given statements are true

Buy Printed Study Material for IBPS, SBI Bank Exams


http://www.bankexamportal.com/study-kit

Page 80

Online Coaching for IBPS, SBI Bank Exams


http://www.bankexamportal.com/elearning
2. According to the passage, for which of the following reasons do the delivery partners fail to serve
their banks goal to expand in the unbanked markets?
(a) They do not have adequate client base to sell their financial products
(b) They do not have adequate knowledge and skills to explain anything beyond basic financial products to
the customers
(c) They do not have the skills to operate advanced technological aids that are a prerequisite to tap the
unbanked market
(1) Only (b) (2) Only (c)
(3) All (a), (b) and (c)
(4) Only (a)
(5) Both (b) and (c)
3. According to the passage, for which of the following reasons is the viability of financial inclusion
under question?
(1) The banks always prefer the cheapest package (to cut cost) while making a choice of technology to be
used
(2) The Business Correspondent Agents are highly demotivated to pursue their activity as a full-time job
(3) The investments made by banks and its delivery partners are not yielding equal amount of returns
(4) The banks do not have adequate number of delivery partners required to tap the unbanked market
(5) The banks do not have adequate manpower to explore the diversity of the unbanked market and thereby
identify the right target customers for various programs
4. In the passage, the number has specified which of the following characteristics of the customer onboarding process?
(1) In involves collection of documents from the applicants in order to validate their details
(2) In involved issuance of smart cards to the customers
(3) If suffers from latency as it takes a long time after submission of documents by the customer
(4) It is an expensive process which people find difficult to afford
(5) All of the given characteristics have been specified
5. What did the author try to highlight in the passage?
(a) The ailing condition of financial inclusion business at present
(b) Strategies that may help bank to expand in the unbanked market
(c) Role of government in modifying the existing financial inclusion policies
(1) Both (a) and (b)
(2) All (a), (b) and (c)
(3) Only (c) (4) Only (a)
(5) Only (b)
6. According to the passage, which of the following ways may help banks to sustain the interest of
their customers after hooking them?

Buy Printed Study Material for IBPS, SBI Bank Exams


http://www.bankexamportal.com/study-kit

Page 81

Online Coaching for IBPS, SBI Bank Exams


http://www.bankexamportal.com/elearning
(a) Adoption of a banking machinist which is not only secure but reassuring to the customers
(b) Increasing the number of delivery partners in rural market
(c) Introduction of a simple and intuitive user application
(1) Only (a) (2) Only (c)
(3) Only (b) (4) All (a), (b) and (c)
(5) Both (a) and (c)
Directions: Choose the word which is most similar in meaning to the word printed in bold as used in the
passage.
7. Multitude
(1) Impoverished (2) Handful
(3) Acknowledged (4) Plenty
(5) Solitude
8. Ubiquitous
(1) Quintessential
(2) Popular
(3) Omnipresent
(4) Simplified
(5) Abnormal
Directions: Choose the word which is most opposite in meaning to the word printed in bold as used in the
passage.
9. Dormant
(1) Emaciated (2) Pertinent
(3) Cornered (4) Rejected
(5) Active
10. Delayed
(1) Perturbed (2) Popularized
(3) Expedited (4) Stablised
(5) Repressed
Directions: Read the following passage carefully and answer the question given below it. Certain words have
been printed in bold to help you locate them while answering some of the questions.
The evolution of Bring Your Own Device (BYOD) trend has been as profound as it has been rapid. It
represents the more visible sign that the boundaries between personal life and work life are blurring. The 9am
5pm model of working solely from office has become archaic and increasingly people are working extended
hours from a range of locations. At the very heart of this evolution is the ability to access enterprise networks
from anywhere and anytime. The concept of cloud computing serves effectively to extend the office out of

Buy Printed Study Material for IBPS, SBI Bank Exams


http://www.bankexamportal.com/study-kit

Page 82

Online Coaching for IBPS, SBI Bank Exams


http://www.bankexamportal.com/elearning
office. The much heralded benefit of BYOD is greater productivity. However, recent research has suggested
that this is the greatest myth of BYOD and the reality is that BYOD in practice poses new challenges that may
outweigh the benefits. A worldwide survey commissioned by Fortieth chose to look at attitudes towards
BYOD and security from the users point of view instead of the IT managers. Specifically the survey was
conducted in 15 territories on groups of graduate employees in the early twenties because they represent the
first generation to enter the workplace with an expectation of own device use. Moreover, they also represent
tomorrows influences and decision makers. The survey findings reveal that for financial organizations, the
decision to embrace BYOD is extremely dangerous. Larger organisations will have mature IT strategies and
policies in place. But what about smaller financial business? They might not have such well developed
strategies to protect confidential data.
Crucially, within younger employee group, 55% of the people share an expectation that they should be allowed
to use their own devices in workplace or for work purposes. With this expectation comes the very real risk
that employees may consider contravening company policy banning the use of won devices. The threats posed
by this level of subversion cannot be overstated. The survey casts doubt on the idea of BYOD leading to
greater productivity by revealing the real reason people want to use their own devices. Only 26% of people in
this age group cite efficiency as the reason they want to use their own devices, while 63% admit that the main
reason is so they have access to their favourite applications. But with personal applications so close to hand,
the risks to the business must surely include distraction and time wasting. To support this assumption 46%
people polled acknowledged time wasting as the greatest threat to the organisation, while 42% citing greater
exposure to theft or loss of confidential data. Clearly, from a user perspective there is great deal of
contradiction surrounding BOYD and there exists an undercurrent of selfishness where users expect to use
their own devices, but mostly for personal interest. They recognize the risks to the organisation but are adamant
that hose risks are worth taking.
11. According to the passage, for which of the following reasons did Fortieth conduct the survey of a
groups of graduate employees in their early twenties?
(a) As this group represents the future decision makers
(b) As this group represents the first generation who entered the workforce with a better understanding of
sophisticated gadgets
(c) As this group represents the first generation to enter the workplace expecting that they can use their own
device for work purpose
(1) All (a), (b) and (c)
(2) Only (c) (3) Both (a) and (c)
(4) Only (a) (5) Only (b)
12. Which of the following is not true about BOYD?
(1) BOYD enables employees to access enterprise network from anywhere and anytime
(2) Due to evolution of BOYD trend the 9am 5 pm model of working solely from office has become
outdated
(3) Recent research has confirmed that BOYD boosts organizational productivity
(4) The concept of cloud computing facilitates the BOYD trend
(5) All the given facts are true
13. According to the passage, why would the decision to embrace BYOD prove dangerous to smaller
financial businesses?

Buy Printed Study Material for IBPS, SBI Bank Exams


http://www.bankexamportal.com/study-kit

Page 83

Online Coaching for IBPS, SBI Bank Exams


http://www.bankexamportal.com/elearning
(1) Their employees have poor knowledge about their devices which in turn pose a threat to the confidential
data of the organisation
(2) The employees are more vulnerable to misplacement of device
(3) They mace lack mature IT strategies and policies required to protect confidential data
(4) They cannot afford to deal with damage liability issues of employee-owned devices
(5) Their employees have a tendency to change jobs frequently
14. According to the passage, the expectation of younger employees that they should be allowed to use
their own devices in the workplace, entails which of the following risks?
(a) Younger employees may deliberately transfer confidential data of their companies to rivals if they are not
allowed to use their own devices for work purpose.
(b) Younger employees may strongly feel like leaving the company if it prevents usages of own device and
join some other company that does not have such stringent policies
(c) Younger employees may consider flouting company policy prohibiting usage of their own devices in the
workplace or for work purposes
(1) Only (c) (2) Only (b)
(3) Both (a) and (c)
(4) Only (a)
(5) All (a), (b) and (c)
15. According to the findings of the survey conducted by Fortinet, why do majority of employees
prefer using their own devices for work purpose?
(1) As they often find that the devices provided by the company lack quality
(2) As they have access to their favourite applications while working
(3) As majority of them believe that output doubles when they use their own devices for work purpose
(4) As handling data from their own devices reinforces their sense of responsibility
(5) As it helps them create a brand of their own
16. What is/are the authors main objective (s) in writing the passage?
(a) To break the myth that BOYD promotes employee efficiency and organizational productivity
(b) To suggest ways to control subversion across levels of corporate chain of command
(c) To throw light upon the fact that employees even after knowing the risks involved, prefer to use their
own device for work purpose mostly for personal benefits
(1) Both (a) and (a)
(2) All (a), (b) and (c)
(3) Only (c)
(4) Only (a)
(5) Only (b)
Directions: Choose the word which is most similar in meaning to the word printed in bold as used in the
passage.
17. Heralded

Buy Printed Study Material for IBPS, SBI Bank Exams


http://www.bankexamportal.com/study-kit

Page 84

Online Coaching for IBPS, SBI Bank Exams


http://www.bankexamportal.com/elearning
(1) Suspected (2) Publicized
(3) Dragged (4) Objective
(5) Benefit
18. Outweigh
(1) Control (2) Venture
(3) Perish (4) Determine
(5) Surpass
Directions: Choose the word which is most opposite in meaning to the word printed in bold as used in the
passage.
19. Embrace
(1) Contradict (2) Disobey
(3) Curtail (4) Reject
(5) Obscure
20. Subversion
(1) Compliance (2) Sanity
(3) Popularity (4) Destabilization
(5) Clarity
Directions: Read each sentence to find out whether there is any grammatical mistake/error in it. The error if
any, will be in one part of the sentence. Mark the number of that part with error as your answer. If there is No
error, mark (5).
21. There cannot be any situation where/ (1) somebody makes money in an asset/ (2) located in India
and does not pay tax / (3) either to India or to the country of this origin. / (4) No error (5)
22. India has entered a downward spiral / (1) an aggressive monetary tightening policy / (2) and law
abide sectors are subject to / (3) savage amounts amount of multiple taxes. / (4) No error (5)
23. The bank may have followed / (1) an aggressive monetary tightening policy / (2) but its stated aim
of / (3) curbing inflation have not been achieved. / (4) No error (5)
24. Equal opportunities for advancement / (1) across the length and breadth / (2) of an organisation
will / (3) keep many problems away. / (4) No error (5)
25. A customized data science degree / (1) is yet to become / (2) a standard programme / (3) to Indias
premier educational institutes. / (4) No error (5)
Directions: Each question below has two blanks, each blank indicating that something has been omitted.
Choose the set of words for each blank that best fits the meaning of the sentence as a whole.
26. When you want to digitalise a city ______ with millions, you dont bet _____ the odds.

Buy Printed Study Material for IBPS, SBI Bank Exams


http://www.bankexamportal.com/study-kit

Page 85

Online Coaching for IBPS, SBI Bank Exams


http://www.bankexamportal.com/elearning
(1) proceeding, into
(2) teeming, against
(3) undergoing, adhere
(4) dangling, for
(5) falling, above
27. The numbers ________ by the legitimate online music service providers indicate that a growing
number of users are _______ to buy music.
(1) morphed, ignoring
(2) labeled, thriving
(3) figured, fanatic
(4) painted, interested
(5) touted, willing
28. If India is _________ on protecting its resources, international business appears equality ______ to
safeguard its profit.
(1) dreaded, fragile
(2) stubborn, weak
(3) bent, determined
(4) approaching settled
(5) observed, prepared
29. Brands ________ decision-simplicity strategies make full use of available information to ______
where consumers are on the path of decision making and direct them to the best market offers.
(1) diluting, divulge
(2) tempting, maintain
(3) imputing, overdrive
(4) pursuing, assess
(5) employing, trust
30. Lack of financing options, ________ with HR and technological _______, make small and medium
enterprises sectors the most vulnerable component of our economy.
(1) except, loophole
(2) coupled, challenges
(3) armed, benefits
(4) registered, strategies
(5) strengthened, facilities
Directions: Rearrange the following six sentence (A), (B), (C), (D), (E) and (F) in the proper sequence to
form a meaningful paragraph; then answer the questions given below them.
(a) The group desired to enhance the learning experience in schools with an interactive digital medium that
could be used within and outside the class-room.

Buy Printed Study Material for IBPS, SBI Bank Exams


http://www.bankexamportal.com/study-kit

Page 86

Online Coaching for IBPS, SBI Bank Exams


http://www.bankexamportal.com/elearning
(b) Then the teacher can act on the downloaded data rather than collect it from each and every student and
thereby save his time and effort.
(c) Editor, decided the group of engineers, all alumni of the Indian Institute of technology, when they founded
Edutor Technologies in August 2009.
(d) They can even take tests and submit them digitally using the same tablets and the teachers in turn can
download the tests using the companys cloud services.
(e) With this desire created a solution that digitizes school textbooks and other learning material so that
students no longest to carry as many books to school and back as before, but can access their study material
on their touch-screen tablets.
(f) A mechanic works on motors and an accountant has his computer. Likewise, if a student has to work on a
machine or device, what should it be called?
31. Which of the following sentences should be the FIRST after arrangement?
(1) F (2) D (3) A (4) C
(5) E
32. Which of the following sentences should be the THIRD after arrangement?
(1) A (2) B (3) D (4) D
(5) F
33. Which of the following sentences should be the SIXTH (LAST) after arrangement?
(1) A (2) F 3) E (4) B
(5) D
34. Which of the following sentences should be the FOURTH after arrangement?
(1) A (2) F (3) E (4) B
(5) C
35. Which of the following sentences should be the FIFTH after arrangement?
(1) A (2) D (3) C (4) E
(5) F
Directions: In the following passage there are blanks, each of which has been numbered. These numbers are
printed below the passage and against each, five words/phrases are suggested, one of which first the blank
appropriately. Find out the appropriate word/phrase in each case.
There is a considerable amount of research about the factors that make a company innovate. So is it possible
to create an environment (36) to innovation? This is a particularly pertinent (37) for India today. Massive
problems in health, education etc. (38) be solved using a conventional approach but (39) creative and

Buy Printed Study Material for IBPS, SBI Bank Exams


http://www.bankexamportal.com/study-kit

Page 87

Online Coaching for IBPS, SBI Bank Exams


http://www.bankexamportal.com/elearning
innovative solution that can ensure radical change and (40). There are several factors in Indias (41). Few
countries have the rich diversity that India or its large, young population (42). While these (43) innovation
policy interventions certain additional steps are also required. These include (44) investment in research and
development by (45) the government and the private sector, easy transfer of technology from the academic
word etc. To fulfill its promise of being prosperous and to be at the forefront, India must be innovative.
36. (1) stimuli (2) conducive
(3) incentive (4) facilitated
(5) impetus
37. (1) objective (2) controversy
(3) doubt (4) question
(5) inference
38. (1) cannot (2) possibly
(3) should (4) never
(5) must
39. (1) necessary (2) apply
(3) need (4) consider
(5) requires
40. (1) quantity (2) advantages
(3) increase (4) chaos
(5) growth
41. (1) challenges (2) praises
(3) favour (4) leverage
(5) esteem
42. (1) blessed
(2) enjoys
(3) endows
(4) prevails
(5) occurs
43. (1) aid
(2) jeopardize
(3) promotes
(4) endure
(5) cater
44. (1) acute
(2) utilizing
(3) restricting
(4) inspiring
(5) increased

Buy Printed Study Material for IBPS, SBI Bank Exams


http://www.bankexamportal.com/study-kit

Page 88

Online Coaching for IBPS, SBI Bank Exams


http://www.bankexamportal.com/elearning
45. (1) both
(2) besides
(3) combining
(4) participating
(5) also
Directions: In each of the following question for words are given of which two words are most nearly the
same or opposite in meaning. Find the two words which are most nearly the same or opposite in meaning and
indicate the number of the correct letter combination, by darkening the appropriate oval in your answer sheet.
46. (a) Consent
(b) Nascent
(c) Emerging
(d) Insecure
(1) A C
(2) B D
(3) B - C
(4) A D
(5) A B
47. (a) Elated
(b) Eccentric
(c) Explicit
(d) Abnormal
(1) A B
(2) B D
(3) A C
(4) A D
(5) D C
48. (a) Abundance (b) Incomparable
(c) Projection (d) Plethora
(1) A C (2) A B
(3) C D (4) B D
(5) A D
49. (a) Purposefully (b) Inaccurately
(c) Inadvertently (d) Unchangeably
(1) A C (2) A B
(3) B C (4) B D
(5) A D
50. (a) germane (b) generate
(c) reliable (d) irrelevant

Buy Printed Study Material for IBPS, SBI Bank Exams


http://www.bankexamportal.com/study-kit

Page 89

Online Coaching for IBPS, SBI Bank Exams


http://www.bankexamportal.com/elearning
(1) B D
(2) B C
(3) A B
(4) C D
(5) A D

Buy Printed Study Material for IBPS, SBI Bank Exams


http://www.bankexamportal.com/study-kit

Page 90

Online Coaching for IBPS, SBI Bank Exams


http://www.bankexamportal.com/elearning

General Awareness, Marketing and Computer


1. Every year March 20 is celebrated as what day ?
(A) World Sparrow Day
(B) International Women's Day
(C) World Cuckoo Day
(D) International Child Day
(E) International Mother's Day

2. Invisible Export means export of


(A) Services
(B) Prohibited goods
(C) Restricted goods
(D) Goods as per OGL list
(E) Other than those given as options

3. One of the famous Indian Sportsperson released his/her autobiography'Playing to Win' in


November 2012. Name the sports-person correctly
(A) Saina Nehwal
(B) Mary Kom
(C) Yuvraj Singh
(D) Sachin Tendulkar
(E) Sushil Kumar

4. The NRIs while investing in the equity of a company cannot buy more than prescribed percentage
of the paid up capital of that company .What is the prescribed percentage ?
(A) 2%
(B) 3%
(C) 4%
(D) 5%
(E) 6%

5. Government as part of the recapitalisation plan, infused capital in State Bank of India recently.
Indicate the approximate capital infusion
(A) Rs. 500 cr.
(B) Rs. 1000 cr.
(C) Rs. 1,500 cr.
(D) Rs. 2,000 cr.
(E) Rs. 3,000 cr.

6. Prof. Muhammad Yunus, the recipient of the Nobel Peace Prize 2006 is the exponent of which of the
following concepts in the field of banking ?
(A) Core Banking

Buy Printed Study Material for IBPS, SBI Bank Exams


http://www.bankexamportal.com/study-kit

Page 91

Online Coaching for IBPS, SBI Bank Exams


http://www.bankexamportal.com/elearning
(B) Micro Credit
(C) Retail Banking
(D) Real Time Gross Settlement
(E) Internet Banking

7. Banks in India are required to maintain a portion of their demand and time liabilities with the
Reserve Bank of India. This portion is called
(A) Statutory Liquidity Ratio
(B) Cash Reserve Ratio
(C) Bank Deposit
(D) Reverse Repo
(E) Government Securities

8. The European Union has adopted which of the following as a common currency ?
(A) Dollar
(B) Dinar
(C) Yen
(D) Euro
(E) Peso

9. Who is the Captain of Australian Cricket Team, which currently (March 2013) visited India ?
(A) Michael Clarke
(B) Shane Watson
(C) Shane Warne
(D) Michael Hussey
(E) Ricky Ponting

10. Who is the author of the book 'Women of Vision' ?


(A) Ravinder Singh
(B) Preeti Shenoy
(C) Amish Tripathi
(D) Durjoy Dutta
(E) Alam Srinivas

11. Which of the following term is associated with the game of Cricket ?
(A) Bouncer
(B) Love
(C) Goal
(D) Mid Court
(E) Collision

12. The Holidays for the Banks are declared as per


(A) Reserve Bank Act

Buy Printed Study Material for IBPS, SBI Bank Exams


http://www.bankexamportal.com/study-kit

Page 92

Online Coaching for IBPS, SBI Bank Exams


http://www.bankexamportal.com/elearning
(B) Banking Regulation Act
(C) Negotiable Instruments Act
(D) Securities and Exchange Board of India Act
(E) Companies Act

13. In banking business, when the borrowers avail a Term Loan, initially they are given a repayment
holiday and this is referred as
(A) Subsidy
(B) Interest Water
(C) Re-phasing
(D) Interest concession
(E) Moratorium

l4. One of IT Company from India has become the first Indian Company to trade on NYSE Euronext
London and Paris markets from February 2013 onwards. Which is this company ?
(A) Wipro Infotech Ltd.
(B) L&T Infotech
(C) HCL Technologies LTD.
(D) Infosys Technologies Limited
(E) Polaris Financial Technology Ltd.

l5. Banking Ombudsman is appointed by


(A) Government of India
(B) State Governments
(C) RBI
(D) ECGC
(E) Exim Bank

l6. BSE (Bombay Stock Exchange), the oldest stock exchange in Asia has joined hands with one more
International index in February 2013. This association has resulted in change of name of BSE index.
What is the change of name effected ?
(A) Dow Jones BSE Index
(B) NASDAQ BSE Index
(C) S&P BSE Index
(D) Euronext BSE Index
(E) Other than those given as options

17. Interest on Savings deposit now-a-days is


(A) Fixed by RBI
(B) Fixed by the respective Banks
(C) Fixed by the Depositors
(D) Fixed as per the contract between Bank and the Consumer Court
(E) Not pay by the Bank

Buy Printed Study Material for IBPS, SBI Bank Exams


http://www.bankexamportal.com/study-kit

Page 93

Online Coaching for IBPS, SBI Bank Exams


http://www.bankexamportal.com/elearning
18. The customers' by opening and investing in the Tax Saver Deposit Account Scheme in a bank
would get benefit under
(A) Sales Tax
(B) Customs Duty
(C) Excise Duty
(D) Professional Tax
(E) Income Tax

19. Pre-shipment finance is provided by the banks only to


(A) Credit Card Holders
(B) Students aspiring for further studies
(C) Brokers in equity market
(D) Village Artisans
(E) Exporters

20. A non-performing asset is


(A) Money at call and short notice
(B) An asset that ceases to generate income
(C) Cash balance in till
(D) Cash balance with RBI
(E) Balance with other banks

21. Interest below which a bank is not expected to lend to customers is known as
(A) Deposit Rate
(B) Base Rate
(C) Prime Lending Rate
(D) Bank Rate
(E) Discount Rate

22. Government usually classifies its expenditure in term of planned and non-planned expenditure.
Identify, which is the correct definition of Planned expenditure ?
(A) It represents the expenditure of all the State Governments
(B) It represents the total expenditure of the Central Government
(C) It is the expenditure which is spent through centrally sponsored programmes and flagship schemes of
Government
(D) It represents the expenditure incurred on Defence
(E) Other than those given as options

23. Which of the following organization is made specifically responsible for empowering Micro, Small
and Medium enterprises in India ?
(A) NABARD
(B) RBI
(C) SIDBI
(D) ECGC

Buy Printed Study Material for IBPS, SBI Bank Exams


http://www.bankexamportal.com/study-kit

Page 94

Online Coaching for IBPS, SBI Bank Exams


http://www.bankexamportal.com/elearning
(E) SEBI

24. The Union Budget for 2013-14 proposed by the Finance Minister on February 28, 2013 announced
introduction of new variety of bonds by the Government. What is the name of these bonds ?
(A) Deep discount bonds
(B) Zero Coupon bonds
(C) Bullet Bonds
(D) Inflation Indexed Bonds
(E) Inflation Variable Bonds

25. RBI has released its guidelines for entry of new banks in the private sector in the month of
February 2013. One of the norms is at least a prescribed percentage of branches of new bank should
be set in unbanked rural centres with a population of upto 9,999 people. What is the percentage of
such branches prescribed in the norms?
(A) 10%
(B) 15%
(C) 18%
(D) 25%
(E) Other than those given as options

26. A joystick is primarily used to/for


(A) Control sound on the screen
(B) Computer gaming
(C) Enter text
(D) Draw pictures
(E) Print text

27. The CPU comprises of Control, Memory and .. units.


(A) Microprocessor
(B) Arithmetic/Logic
(C) Output
(D) ROM
(E) Input

28. Which of the following uses a handheld operating system ?


(A) A super computer
(B) A personal computer
(C) A laptop
(D) A mainframe
(E) A PDA

29. To display the contents of a folder in Windows Explorer you should


(A) click on it

Buy Printed Study Material for IBPS, SBI Bank Exams


http://www.bankexamportal.com/study-kit

Page 95

Online Coaching for IBPS, SBI Bank Exams


http://www.bankexamportal.com/elearning
(B) collapse it
(C) name it
(D) give it a password
(E) rename it

30. 'C' in CPU denotes


(A) Central
(B) Common
(C) Convenient
(D) Computer
(E) Circuitry

31. .. is the most important/powerful computer in a typical network.


(A) Desktop
(B) Network Client
(C) Network server
(D) Network station
(E) Network switch

32. Which is not a storage device ?


(A) A CD
(B) A DVD
(C) A Floppy disk
(D) A Printer
(E) A Hard disk

33. Which bf the following refers to the rectangular area for displaying information and running
programs ?
(A) Desktop
(B) Dialog box
(C) Menu
(D) Window
(E) Icon

34. .. devices convert human-understandable data and programs into a form that the
computer can process.
(A) Printing
(B) Output
(C) Solid state
(D) Monitor
(E) Input

Buy Printed Study Material for IBPS, SBI Bank Exams


http://www.bankexamportal.com/study-kit

Page 96

Online Coaching for IBPS, SBI Bank Exams


http://www.bankexamportal.com/elearning
35. The software that is used to create text-based documents are referred to as
(A) DBMS
(B) suites
(C) spreadsheets
(D) presentation software
(E) word Processors

36. .. is a Windows utility program that locates and eliminates unnecessary fragments and
rearranges files and unused-disk space to optimize operations.
(A) Backup
(B) Disk Cleanup
(C) Disk Defragmenter
(D) Restore
(E) Disk Restorer

37. Which of the following refers to too much electricity and may cause a voltage surge?
(A) Anomaly
(B) Shock
(C) Spike
(D) Virus
(E) Splash

38. A (n) .. appearing on a web page opens another document when clicked.
(A) anchor
(B) URL
(C) hyperlink
(D) reference
(E) heading

39. A successful marketing Person requires one of the following qualities


(A) Empathy
(B) Sympathy
(C) Insistence
(D) Aggressiveness
(E) Pride

40. Market segmentation can be resorted to by dividing the target group as per
(A) Income levels of customers
(B) Age of the employees
(C) Needs of the sales persons
(D) Marketing skills of the employees
(E) Size of the Organisation

Buy Printed Study Material for IBPS, SBI Bank Exams


http://www.bankexamportal.com/study-kit

Page 97

Online Coaching for IBPS, SBI Bank Exams


http://www.bankexamportal.com/elearning
41. Innovation in marketing is same as
(A) Abbreviation
(B) Communication
(C) Creativity
(D) Aspiration
(E) Research work

42. Effective Selling Skills depends on


(A) Number of languages known to the DSA
(B) Data on marketing staff
(C) Information regarding IT market
(D) Knowledge of related market
(E) Ability to talk fast

43. A Direct Selling Agent (DSA) is required to be adept in


(A) Surrogate marketing
(B) Training Skills
(C) Communication skills
(D) Market Research
(E) OTC Marketing

44. Leads can be best sourced from


(A) Foreign customers
(B) Yellow pages
(C) Dictionary
(D) List of vendors
(E) Local supply chains

45. Value added services means


(A) Low cost products
(B) High cost products
(C) At par services
(D) Additional services for the same cost
(E) Giving Discounts

46. Post-sales activities includes


(A) Sales presentation
(B) Customer Feedback
(C) Customer indentification
(D) Customer apathy
(E) Product design

Buy Printed Study Material for IBPS, SBI Bank Exams


http://www.bankexamportal.com/study-kit

Page 98

Online Coaching for IBPS, SBI Bank Exams


http://www.bankexamportal.com/elearning
47. The target market for Debit Cards is
(A) All existing account-holders
(B) All agriculturists
(C) All DSAs
(D) All vendors
(E) All outsourced agents

48. The Competitive position of a company can be improved by


(A) increasing the selling price
(B) reducing the margin
(C) ignoring competitors
(D) increasing the cost price
(E) understanding and fulfilling customers needs

49. A good Brand can be built up by way of


(A) Customer grievances
(B) Break-down of IT support
(C) Old age
(D) Large number of products
(E) Consistent offering of good services

50. The USP of a product denotes


(A) Usefulness of the product
(B) Drawback of a product
(C) Main functions
(D) Number of allied products available
(E) High selling features of a product

Buy Printed Study Material for IBPS, SBI Bank Exams


http://www.bankexamportal.com/study-kit

Page 99

Online Coaching for IBPS, SBI Bank Exams


http://www.bankexamportal.com/elearning

SBI PO Exam Paper - 2010


Data Analysis And Interpretation
Direction: Study the following table carefully to answer the questions that follow
Number (n) of candidates (in lakhs) appearing for an entrance examination From six different states
and the percentage (p) of candidates clearing the same over the years

1. What is the respective ratio of total number of candidates clearing the entrance exam from State B
in the year 2004 to those clearing the entrance exam from State C in the same year
(1) 221: 148 (2) 218: 143
(3) 148: 221 (4) 143: 218
(5) None of these
2. In which year did the highest number of candidates clear the entrance exam from State D ?
(1) 2008 (2) 2006
(3) 2009 (4) 2007
(5) None of these
3. What is the number of candidates not clearing the entrance exam from State A in the year 2007?
(1) 186820 (2) 11682
(3) 1868200 (4) 116820
(5) None of these
4. What is the total numberof candidates clearing the entrance exam from States E and F together in
the year 2006 ?
(1) 16160 (2) 110660
(3) 1.1066 (4) 1106600
(5) None of these

Buy Printed Study Material for IBPS, SBI Bank Exams


http://www.bankexamportal.com/study-kit

Page 100

Online Coaching for IBPS, SBI Bank Exams


http://www.bankexamportal.com/elearning
5. What is the average number of candidates appearing for the entrance exam from State D in the
years 2007, 2008 and 2009 together ?

Direction: Study the given information carefully and answer the questions that follow
An urn contains 6 red, 4 blue, 2 green and 3 yellow marbles.
6. If four marbles are picked at random, what is the probability that at least one is blue ?
(1) 4/11 (2) 69/91 (3) 11/15
(4) 22/91 (5) None of these
7. If two marbles are picked at random, what is the probability that both are red ?
(1) 1/6 (2) 1/3 (3) 2/15
(4) 2/5 (5) None of these
8. If three marbles are picked at random, what is the probability that two are blue and one is yellow?
(1) 3/91 (2) 1/5 (3) 18/455
(4) 7/15 (5) None of these
9. If four marbles are picked at random, what is the probability that one is green, two are blue and
one is red ?
(1) 24/55 (2) 13/55 (3) 11/15
(4) 7/91 (5) None of these
10. If two marbles are picked at random, what is the probability that either both are green or both are
yellow ?
(1) 5/91 (2) 1/35 (3) 1/3
(4) 4/105 (5) None of these
Direction: Study the given pie-charts carefully to answer the questions that follow
Breakup of number of employees working in different departments of an organisation, the, number of
males and the number of employees who recently got promoted in each department break-up of
employees working in different departments total number of employees = 3,600 Employees working in
different departments:

Buy Printed Study Material for IBPS, SBI Bank Exams


http://www.bankexamportal.com/study-kit

Page 101

Online Coaching for IBPS, SBI Bank Exams


http://www.bankexamportal.com/elearning

Break-up of number of males in each department


Total number of males in the organisation = 2,040
Break-up of number of males working in each department

Break-up of number of employees who recently got promoted in each department


Total number of employees who got promoted = 1,200
Number of employees who recently got promoted from each department

Buy Printed Study Material for IBPS, SBI Bank Exams


http://www.bankexamportal.com/study-kit

Page 102

Online Coaching for IBPS, SBI Bank Exams


http://www.bankexamportal.com/elearning
11. If half of the number of employees who got promoted from the IT department were males, what
was the approximate percentage of males who got promoted from the IT department ?
(1) 61 (2) 29 (3) 54
(4) 42 (5) 38
12. What is the total number of females working in the Production and Marketing departments
together ?
(1) 468 (2) 812 (3) 582
(4) 972 (5) None of these
13. How many females work in the Accounts department ?
(1) 618 (2) 592 (3) 566
(4) 624 (5) None of these
14. The total number of employees who got promoted from all the departments together was what
percent of the total number of employees working in all the departments together ? (Rounded off to
the nearest integer)
(1) 56 (2) 21 (3) 45
(4) 33 (5) 51
15. The number of employees who got promoted from the HR department was what percent of the
total number of employees working in that department ? (rounded off to two digits after decimal)
(1) 36.18 (2) 30.56 (3) 47.22
(4) 28.16 (5) None of these
Direction: Study the graph carefully to answer the questions that follow

Buy Printed Study Material for IBPS, SBI Bank Exams


http://www.bankexamportal.com/study-kit

Page 103

Online Coaching for IBPS, SBI Bank Exams


http://www.bankexamportal.com/elearning
16. If the profit earned by Company L in the year 2005 was Rs. 1.84 lakhs, what was the profit earned
by the company in the year 2006 ?
(1) Rs. 2.12 lakhs (2) Rs. 2.3 lakhs
(3) Rs. 2.04 lakhs
(4) Cannot be determined
(5) None of these
17. If the profit earned by Company M in the year 2008 was Rs. 3.63 lakhs, what was he amount of
profit earned by it in the year 2006 ?
(1) Rs. 2.16 lakhs (2) Rs.1.98 lakhs
(3) Rs. 2.42 lakhs
(4) Cannot be determined
(5) None of these
18. What is the average percent rise in profit of Company L over all the years together ?

19. Which of the following statements is TRUE with respect to the above graph ?
(1) Company M made the highest profit in the year 2009
(2) Company L made least profit in the year 2005
(3) The respective ratio between the profits earned by Company L and M In the year 2006 was 6:5
(4) Company L made the highest profit in the year 2008
(5) All are true
20. What is the percentage increase in percent rise in profit of Company M in the year 2009 from the
previour year ?
(1) 25 (2) 15 (3) 50
(4) 75 (5) None of these
Direction: Study th a information carefully to answer the questions that follow.
A school consisting of a total of 1560 students has boys and girls in the ratio of 7:5 respectively. All the
students are enrolled in different types of hobby classes, viz: Singing, Dancing and Painting. One-fifth of the
boys are enrolled in only Dancing classes. Twenty percent of the girls are enrolled in only Painting classes.
Ten percent of the boys are enrolled in only Singing classes. Twenty four percent of the girls are enrolled in
both Singing and Dancing classes together. The number of girls enrolled in only Singing classes is two
hundred percent of the boys enrolled in the same. One-thirteenth of the boys are enrolled in all the three
classes together. The respective ratio of boys enrolled in Dancing and Painting classes together to the girls
enrolled in the same is 2 :1 respectively. Ten percent of the girls are enrolled in only Dancing classes

Buy Printed Study Material for IBPS, SBI Bank Exams


http://www.bankexamportal.com/study-kit

Page 104

Online Coaching for IBPS, SBI Bank Exams


http://www.bankexamportal.com/elearning
whereas eight percent of the girls are enrolled in both Dancing and Painting classes together. The remaining
girls are enrolled in all the three classes together. The number of boys enrolled in Singing and Dancing
classes together is fifty percent of the number of girls enrolled in the same. The remaining boys are enrolled
in only Painting classes.
21. What is the total number of boys who are enrolled in Dancing ?
(1) 318 (2) 364
(3) 292 (4) 434
(5) None of these
22. Total number of girls enrolled in Singing is approximately what percent of the total number of
students in the school ?
(1) 37 (2) 19
(3) 32 (4) 14
(5) 26
23. What is the total number of students enrolled in all the three classes together ?
(1) 135 (2) 164
(3) 187 (4) 142
(5) None of these
24. Number of girls enrolled in only Dancing classes is what percent of the boys enrolled in the same?
(rounded off to two digits after decimal)
(1) 38.67 (2) 35.71
(3) 41.83 (4) 28.62
(5) None of these
25. What is the respective ratio of the number of girls enrolled in only Painting classes to the number
of boys enrolled in the same ?
(1) 77 : 26 (2) 21 : 73
(3) 26 : 77 (4) 73 : 21
(5) None of these
Direction: Study the table carefully to answer the questions that follow.

Buy Printed Study Material for IBPS, SBI Bank Exams


http://www.bankexamportal.com/study-kit

Page 105

Online Coaching for IBPS, SBI Bank Exams


http://www.bankexamportal.com/elearning

26. What is the respective ratio between the profit earned by shopkeeper U in the months February2010 and March - 2010 together to that earned by shopkeeper Q in the same months ?
(1) 637: 512 (2) 621 : 508
(3) 512: 637 (4) 508: 621
(5) None of these
27. What is the percent increase in profit of shopkeeper S in the month of December - 2009 over the
previous month ? (rounded off to two digits after decimal)
(1) 3.15 (2) 2.67
(3) 2.18 (4) 3.33
(5) None of these
28. Which shopkeepers profit kept increasing continuously over the given months ?
(1) R (2) Q (3) T
(4) U (5) None of these
29. What is the difference in profit earned by shopkeeper T in January - 2010 from the previous
month ?
(1) Rs. 640/- (2) Rs. 420/(3) Rs. 380/- (4) Rs. 760/(5) None of these
30. What was the average profit earned by shopkeeper R in the months of October 2009 and
November 2009 together ?
(1) 5405 (2) 5040 (3) 4825
(4) 4950 (5) None of these
Direction: Study the given graph carefully to answer the questions that follow
Number of days taken by three carpenters to finish making one piece each of four different items of
furniture

Buy Printed Study Material for IBPS, SBI Bank Exams


http://www.bankexamportal.com/study-kit

Page 106

Online Coaching for IBPS, SBI Bank Exams


http://www.bankexamportal.com/elearning

31. If Carpenter X and Carpenter Y were to make a chair together how many days would they take?
(1) 1 day (2) 4 days (3) 3 days
(4) 2 days (5) None of these
32. If Carpenters X, Y and Z were to make a table together how many days would they take ?
(1) 4 days (2) 3 days (3) 1 day
(4) 2 days (5) None of these
33. What is the total number of days that Carpenter Z will take to make one piece each of all the four
items together ?
(1) 32 days (2) 24 days
(3) days (4) days
(5) None of these
34. The radius of a circular field is equal to the side of a square field whose perimeter is 784 feet.
What is the area of the circular field ?
(1) 107914 Sq.ft (2) 120736 Sq.ft.
(3) 107362 Sq.ft. (4) 127306 Sq.ft.
(5) None of these
35. In how many different ways can the letters of the word STRESS be arranged
(1) 360 (2) 240
(3) 720
(4) 120
(5) None of these
Direction: Study the given table carefully to answer the questions that follow

Buy Printed Study Material for IBPS, SBI Bank Exams


http://www.bankexamportal.com/study-kit

Page 107

Online Coaching for All IBPS Examination


What You will get:
1. All the relevant and required materials of subjects mention in the syllabus like:

100% IBPS Exam Syllabus Covered with MCQs.

Quantitative Aptitude

Reasoning Ability

General English

Hindi Language

General Awareness

Computer Knowledge

2. Special Material based on Current Affair for IBPS Examination


3. Online Tests for Practice will be conducted.

For More Information Click Given below link:


http://bankexamportal.com/online-coaching/ibps-exams

Online Coaching for IBPS, SBI Bank Exams


http://www.bankexamportal.com/elearning
Number of people staying in five different localities and the percentage breakup of men, women and
children in them
LOCALITY TOTAL NO. PERCENTAGE OF PEOPLE MEN WOMEN CHILDREN

36. Total number of people staying in locality J forms approximately what percent of the total number
of people staying in locality F ?
(1) 81 (2) 72 (3) 78
(4) 93 (5) 87
37. What is the total number of children staying in localities H and I together ?
(1) 1287 (2) 1278 (3) 1827
(4) 1728 (5) None of these
38. The number of women staying in which locality is the highest ?
(1) H (2) J (3) F
(4) G (5) None of these
39. What is the total number of men and children staying in locality I together
(1) 4115 (2) 4551 (3) 4515
(4) 4155 (5) None of these
40. What is the respective ratio of number of men staying in locality F to the number of men staying in
locality H ?
(1) 517: 416 (2) 403: 522
(3) 416: 517 (4) 522: 403
(5) None of these
41. The compound interest earned by Suresh on a certain amount at the end of two years at the rate of
8 p.c.p.a was Rs. 1,414.4. What was the total amount that Suresh got back at the end of two years in
the form of principal plus interest earned?
(1) Rs. 9,414.4 (2) Rs. 9,914.4
(3) Rs. 9,014.4 (4) Rs. 8,914.4
(5) None of these

Buy Printed Study Material for IBPS, SBI Bank Exams


http://www.bankexamportal.com/study-kit

Page 108

Online Coaching for IBPS, SBI Bank Exams


http://www.bankexamportal.com/elearning
42. The respective ratio of the present ages of a mother and daughter is 7 : 1. Four years ago the
respective ratio of their ages was 19:1. What will be the mothers age four years from now ?
(1) 42 years (2) 38 years
(3) 46 years (4) 36 years
(5) None of these
43. Three friends J, K and Ljog around a circular stadium and complete one round in 12, 18 and 20
seconds respectively. In how many minutes will all the three meet again at the starting point
(1) 5 (2) 8 (3) 12
(4) 3 (5) None of these
44. 4 men can complete a piece of work in 2 days. 4 women can complete the same piece of work in 4
days whereas 5 children can complete the same piece of work in 4 days. If, 2 men, 4 women and 10
children work together, in how many days can the work be completed ?
(1) 1 day (2) 3 days (3) 2 days
(4) 4 days (5) None of these
45. The speed of a boat when travelling downstream is 32 Kms. / Hr. , whereas when travelling
upstream it is 28 kms/hr. What is the speed of the boat in still water ?
(1) 27 Kms./Hr. (2) 29 Kms./ Hr.
(3) 31 Kms./ Hr.
(4) Cannot be determined
(5) None of these
Direction: Study the following tables carefully and answer the questions given below
Number of Candidates appeared in a Competitive Examination from five centres over the years
NUMBER

Buy Printed Study Material for IBPS, SBI Bank Exams


http://www.bankexamportal.com/study-kit

Page 109

Online Coaching for IBPS, SBI Bank Exams


http://www.bankexamportal.com/elearning
Approximate-Percentages of candidates qualified to appeared in the-Competitive examination from
five centres over the years
PERCENTAGE

46. In which of the following years was the difference in number of candidates appeared from
Mumbai over the previous year the minimum ?
(1) 2004 (2) 2006 (3) 2007
(4) 2002 (5) None of these
47. In which of the following years was the number of candidates qualified from Chennai, the
maximum among the given years ?
(1) 2007 (2) 2006 (3) 2005
(4) 2003 (5) None of these
48. Approximately what was the total number of candidates qualified from Delhi in 2002 and 2006
together ?
(1) 27250 (2) 25230 (3) 30150
(4) 28150 (5) 26250
49. Approximately how many candidates appearing from Kolkata in 2004 qualified in the competitive
examination ?
(1) 13230 (2) 13540 (3) 15130
(4) 15400 (5) 19240
50. Approximately what was the difference between the number of candidates qualified from
Hyderabad in 2001 and 2002 ?
(1) 1680 (2) 2440 (3) 1450
(4) 2060 (5) 1860

Buy Printed Study Material for IBPS, SBI Bank Exams


http://www.bankexamportal.com/study-kit

Page 110

Online Coaching for IBPS, SBI Bank Exams


http://www.bankexamportal.com/elearning

Reasoning Ability
1. In a certain code language how many goals scored is written as 5 3 9 7; many more matches is
written as 9 8 2 and he scored five is written as 1 6 3. How is goals written in that code language
?
(1) 5 (2) 7 (3) 5 or 7
(4) Data inadequate
(5) None of these
2. In a certain code TEMPORAL is written as OLDSMBSP. How is CONSIDER written in that code?
(1) RMNBSFEJ (2) BNMRSFEJ
(3) RMNBJEFS (4) TOPDQDCH
(5) None of these
3. How many meaningful English words can be made with the letters DLEI using each letter only once
in each word ?
(1) None (2) One (3) Two
(4) Three (5) More than three
4. Among A, B, C, D and E each having different weight, D is heavier than only A and C is lighter
than B and E. Who among them is the heaviest ?
(1) B (2) E (3) C
(4) Data inadequate
(5) None of these
5. Each odd digit in the number 5263187 is substituted by the next higher digit and each even digit is
substituted by the previous lower digit and the digits so obtained are rearranged in ascending order,
which of the following will be the, third digit from the left end after the rearrangement ?
(1) 2 (2) 4 (3) 5
(4) 6 (5) None of these
6. Pratap corrrectly remembers that his mothers birthday is before twenty third April but after
Nineteenth April, whert as his sister correctly remembers that their mothers birthday is not on or
after twenty second April. On which day in April is definitely their mothers birthday ?
(1) Twentieth (2) Twenty-first
(3) Twentieth or twenty-first
(4) Cannot be determined
(5) None of these
7. Ashok started walking towards South. After walking 50 meters he took a right turn and walked 30
meters. He then took a right turn and walked 100 meters. He again took a right turn and walked 30
meters and stopped. How far and in which direction was he from the starting point?

Buy Printed Study Material for IBPS, SBI Bank Exams


http://www.bankexamportal.com/study-kit

Page 111

Online Coaching for IBPS, SBI Bank Exams


http://www.bankexamportal.com/elearning
(1) 50 meters South (2) 150 meters North
(3) 180 meters East (4) 50 meters North
(5) None of these
8. If means +; means ; means and +means ; then 15 8 6 + 12 + 4 = ?
(1) 20 (2) 28 (3)
(4) 2 3 (5) None of these
9. Town D is towards East of town F. Town B is towards North of town D. Town H is towards South of
town B. Towards which direction is town H from town F ?
(1) East (2) South-East
(3) North-East (4) Data inadequate
(5) None of these
10. How many such pairs of letters are there in the word SEARCHES each of which has as many
letters between them in the word as in the English alphabet ?
(1) None (2) One (3) Two
(4) Three (5) More than three
Direction: In each of the questions below are given four statements followed by four conclusions
numbered I, II, III 8s IV. You have to take the given statements to be true even if they seem to be at
given conclusions logically follows from the given statements disregarding commonly known facts.
11. Statements: All cups are bottles. Some bottles are jugs. No jug is plate. Some plates are tables.
Conclusions:
I. Some tables are bottles.
II. Some plates are cups.
III. No table is bottle.
IV. Some jugs are cups.
(1) Only I follows (2) Only II follows
(3) Only III follows (4) Only IV follows
(5) Only either I or III follows
12. Statements: Some chairs are handles. All handles are pots. All pots are mats. Some mats are buses.
Conclusions:
I. Some buses are handles.
II. Some mats are chairs.
III. No bus is handle.
IV. Some mats are handles.

Buy Printed Study Material for IBPS, SBI Bank Exams


http://www.bankexamportal.com/study-kit

Page 112

Online Coaching for IBPS, SBI Bank Exams


http://www.bankexamportal.com/elearning
(1) Only I, II and IV follow
(2) Only II, III and IV follow
(3) Only either I or III and II follow
(4) Only either I or III and IV follow
(5) Only either I or III and II and IV follow
13. Statements: All birds are horses. All horses are tigers. Some tigers are lions. Some lions are monkeys.
Conclusions:
I. Some tigers are horses.
II. Some monkeys are birds.
III. Some tigers are birds.
IV. Some monkeys are horses.
(1) Only I and III follow
(2) Only I, II and III follow
(3) Only II, III and IV follow
(4) All I, II, III and IV follow
(5) None of these
14. Statements: Some benches are walls. All walls are houses. Some houses are jungles. All jungles are
roads.
Conclusions:
I. Some roads are benches.
II. Some jungles are walls.
III. Some houses are benches.
IV. Some roads are houses.
(1) Only land II follow
(2) Only I and III follow
(3) Only III and IV follow
(4) Only II, III and IV follow
(5) None of these
15. Statements: Some sticks are lamps. Some flowers are lamps. Some lamps are dresses. All dresses are
shirts.
Conclusions:
I. Some shirts are sticks.
II. Some shirts are flowers.
III. Some flowers are sticks.
IV. Some dresses are sticks.

Buy Printed Study Material for IBPS, SBI Bank Exams


http://www.bankexamportal.com/study-kit

Page 113

Online Coaching for IBPS, SBI Bank Exams


http://www.bankexamportal.com/elearning
(1) None follows (2) Only I follows
(3) Only II follows (4) Only III follows
(5) Only IV follows
Direction: Study the following information carefully and answer the questions given below:
A, B, C, D, E, F, G and H are eight employees of an organization working in three departments viz.
Personnel, Administration and Marketing with not more than three of them in any department. Each of them
has a different choice of sports from football, cricket, volleyball, badminton, lawn tennis, basketball, hockey
and table tennis not necessarily in the same order.
D works in Administration and does not like either football or cricket. F works in Personnel with only A
who likes table tennis. Eand H do not work in the same department as D. C likes hockey and does not work
in marketing. G does not work in administration and does not like either cricket or badminton. One of those
who work in administration likes football. The one who likes volleyball works in personnel. None of those
who work in administration likes either badminton or lawn tennis. H does not like cricket.
16. Which of the following groups of employees work in Administration department ?
(1) EGH (2) AF (3) BCD
(4) BGD (5) Data inadequate
17. In which department does E work ?
(1) Personnel (2) Marketing
(3) Administration (4) Data inadequate
(5) None of these
18. Which of the following combinations of employee-department-favourite sport is correct?
(1) E Administration Cricket
(2) F Personnel Lawn Tennis
(3) H Marketing Lawn Tennis
(4) B Administration Table Tennis
(5) None of these
19. What is Es favourite sport ?
(1) Cricket (2) Badminton
(3) Basketball (4) Lawn Tennis
(5) None of these
20. What is Gs favourite sport ?
(1) Cricket (2) Badminton
(3) Basketball (4) Lawn Tennis
(5) None of these

Buy Printed Study Material for IBPS, SBI Bank Exams


http://www.bankexamportal.com/study-kit

Page 114

Online Coaching for IBPS, SBI Bank Exams


http://www.bankexamportal.com/elearning
Direction: In the following questions, the symbols # and S are used with the following meaning as
illustrated below.
P $ Q means P is not smaller than Q.
P @ Q means P is neither smaller than nor equal to Q
P # Q means P is neither greater than nor equal to Q.
P d Q means P is neither greater than nor smaller than Q.
P Q means P is not greater than Q.
Now in each of the following questions assuming the given statements to be true, find which of the four
conclusions I, II, III and IV given below them is/are definitely true and give your answer accordingly.
21. Statements: H @ T, T # F, F d E, E V
Conclusions:
I. V $ F
II. E @ T
III. H @ V IV. T # V
(1) Only I, II and III are true
(2) Only I, II and IV are true
(3) Only II, III and IV are true
(4) Only I, III and IV are true
(5) All I, II, III and IV are true
22. Statements: D#R, RK, K@F, F$J
Conclusions:
I. J # R
II. J # K
III. R # F IV. K @ D
(1) Only I, II and III are true
(2) Only II, III and IV are true
(3) Only I, III and IV are true
(4) All I, II, III and IV are true
(5) None of these
23. Statements: N d B, B $ W, W # H, H M
Conclusions :
I. M @ W II. H @ N III. W S N IV. W# N

Buy Printed Study Material for IBPS, SBI Bank Exams


http://www.bankexamportal.com/study-kit

Page 115

Online Coaching for IBPS, SBI Bank Exams


http://www.bankexamportal.com/elearning
(1) Only I is true (2) Only III is true
(3) Only IV is true
(4) Only either III or IV is true
(5) Only either III or IV and I are true
24. Statements: R D, D $ J, J # M, M @ K
Conclusions:
I. K # J II. D @ M
III. R # M
IV. D @ K
(1) None is true (2) Only I is true
(3) Only II is true (4) Only III is true
(5) Only IV is true
25. Statements: M $ K, K @ N, N R, R # W
Conclusions:
I. W @ K II. M $ R III. K @ W
IV. M @ N
(1) Only I and II are true
(2) Only I, II and III are true
(3) Only III and IV are true
(4) Only II, III and IV are true
(5) None of these
Direction: Study the following information carefully and answer the questions given below
Following are the conditions for selecting Senior Manager-Credit in a bank. The candidate must
(i) be a graduate in any discipline with atleast 60 percent marks.
(ii) have post qualification work experience of at least ten years in the Advances Section of a bank.
(iii) be at least 30 years and not more than 40 years as on 1.4.2010. (iv) have secured at least 40 percent
marks in the group discussion. (v) have secured at least 50 percent marks in interview. In the case of a
candidate who satisfies all the conditions EXCEPT
(A) at (i) above but has secured at least 50 percent marks in graduation and at least 60 percent marks in post
graduation in any discipline the case is to be referred to the General Manager Advances.
(B) at (ii) above but has total post qualification work experience of at least seven years out of which at least
three years as Manager-Credit in a bank, the case is to be referred to Executive Director.
In each question below details of one candidate is given. You have to take one of the following courses of
action based on the information provided and the conditions and sub-conditions given above and mark the

Buy Printed Study Material for IBPS, SBI Bank Exams


http://www.bankexamportal.com/study-kit

Page 116

Online Coaching for IBPS, SBI Bank Exams


http://www.bankexamportal.com/elearning
number of that course of action as your answer. You are not to assume anything other than the information
provided in each question. All these cases are given to you as on 01.04.2010.
Give answer
(1) if the case is to be referred to Executive Director.
(2) if the case is to be referred to General Manager-Advances.
(3) if the data are inadequate to take a decision.
(4) if the candidate is not to be selected.
(5) if the candidate is to be selected.
26. Shobha Gupta has secured 50 percent marks in the Interview and 40 percent marks in the Group
Discussion. She has been working for the past eight years out of which four years as Manager-Credit
in a bank after completing her B. A. degree with 60 percent marks. She was born on 12th September
1978.
27. Rohan Maskare was born on 8th March 1974. He has been working in a bank for the past twelve
years after completing his B.Com. degree with 70 percent marks. He has secured 50 percent marks in
both the Group Discussion and the Interview.
28. Prakash Gokhale was born on 4th August 1977. He has secured 65 percent marks in. post
graduation and 58 percent marks in graduation. He has been working for the past ten years in the
Advances Department of a bank after completing his post graduation. He has secured 45 percent
marks in the Group Discussion and 50 percent marks in the Interview.
29. Sudha Mehrotra has been working in the Advances department of a bank for the past twelve years
after completing her B.Com. degree with 60 percent marks. She has secured 50 percent marks in the
Group Discussion and 40 percent marks in the Interview. She was born on 15th February 1972.
30. Amit Narayan was born on 28th May 1974. He has been working in the Advances department of a
bank for the past eleven years after completing his B.Sc. degree with 65 percent marks. He has
secured 55 percent marks in the Group discussion and 50 percent marks in the interview.
Direcion: In each question below is given a statement followed by three courses of action numbered (A),
(B) and (C). A course of action is a step or administrative decision to be taken for. improvement, follow-up
or further action in regard to the problem, policy, etc. On the basis of the information given in the statement,
you have to assume everything in the statement to be true, then decide which of the suggested courses of
action logically follow(s) for pursuing.
31. Statement : A heavy unseasonal downpour during tho last two days has paralysed the, normal life in the
state in which five persons were killed but this has provided a huge relief to the problem of acute water crisis
in the state.
Courses of action:
(A) The state government should set up a committee to review the alarming situation.
(B) The state government should immediately remove all the restrictions, on use of potable water in all the

Buy Printed Study Material for IBPS, SBI Bank Exams


http://www.bankexamportal.com/study-kit

Page 117

Online Coaching for IBPS, SBI Bank Exams


http://www.bankexamportal.com/elearning
major cities in the state.
(C) The state government should send relief supplies to all the affected areas in the state.
(1) None (2) Only (A)
(3) Only (B) and (C) (4) Only (C)
(5) All (A), (B), (C)
32. Statement: A large private bank has decided to retrench one-third of its employees in view of the huge
losses incurred by it during the past three quarters.
Courses of action:
(A) The Govt. should issue a notification to general public to immediately: stop all transactions with the
bank.
(B) The Govt. should direct the bank to refrain from retrenching its employees.
(C) The Govt. should ask the central bank of the country to initiate an enquiry into the banks activities and
submit its report.
(1) None (2) Only (A)
(3) Only (B) (4) Only (C)
(5) Only (A) and (C)
33. Statement: Many political activists have decided to stage demonstrations and block traffic movement in
the city during peak hours to protest against the steep rise in prices of essential commodities.
Courses of action:
(A) The Govt. should immediately ban all forms of agitations in the country.
(B) The police authority of the city should deploy additional forces all over the city to help traffic movement
in the city.
(C) The state administration should carry out preventive arrests of the known criminals staying in the city.
(1) Only (A) (2) Only (B)
(3) Only (C) (4) Only (A) and (B)
(5) None of these
34. Statement: The school dropout rate in many districts in the state has increased sharply during the last
few years as the parents of these children make them work in the fields owned by others to earn enough for
them to get at least one meal a day.
Courses of action:
(A) The Govt. should put up a mechanism to provide foodgrains to the poor people in these districts through
public distribution system to encourage the parents to send their wards to school.
(B) The Govt. should close down some of these schools in the district and deploy the teachers of these
schools to nearby schools and also ask remaining students to join these schools.
(C) Govt. should issue arrest warrants for all the parents who force their children to work in fields instead of
attending classes.

Buy Printed Study Material for IBPS, SBI Bank Exams


http://www.bankexamportal.com/study-kit

Page 118

Online Coaching for IBPS, SBI Bank Exams


http://www.bankexamportal.com/elearning
(1) Only (A) (2) Only (B)
(3) Only (C) (4) Only (A) and (B)
(5) None of these
35. Statement: One aspirant was killed due to stampede while participating in a recruitment drive of police
constables.
Courses of action:
(A) The officials incharge of the recruitment process should immediately be suspended.
(B) A team of officials should be asked to find out the circumstances which led to the death of the aspirant
and submit its report within a week.
(C) The Govt. should ask the home department to stagger the number of aspirants over more number of days
to avoid such incidents in future.
(1) Only (A) (2) Only (B)
(3) Only (C) (4) Only (B) and (C)
(5) None of these
36. Effect: Majority of the employees of the ailing organization opted for voluntary retirement scheme and
left the organization with all their retirement benefits within a fortnight of launching the scheme.
Which of the following can be a probable cause of the above effect ?
(1) The company has been making huge losses for the past five years and is unable to pay salary to its
employees in time.
(2) The management of the company made huge personal gains through unlawful activities.
(3) One of the competitors of the company went bankrupt last year.
(4) The company owns large tracts of land in the state which will fetch huge sum to its owners.
(5) None of these
37. Statement: Most of the companies in IT and ITES sectors in India have started hiring from engineering
college campuses this year and are likely to recruit much more than yearly recruitment of the earlier years.
Which of the following substantiates the facts stated in the above statement ?
(1) IT and ITES are the only sectors in India which are hiring from engineering college campuses.
(2) Govt. has stepped up recruitment activities after a gap of five years.
(3) The IT and ITES companies have now decided to vi sit the engineering college campuses for tier II cities
in India as well.
(4) Availability of qualified engineers will substantially in crease in the near future.
(5) None of these
38. Cause: The Govt. has recently increased its taxes or, petrol and diesel by about 10 percent. Which of the
following can be a possible effect of the above cause ?
(1) The petroleum companies will reduce the prices of petrol and diesel by about 10 percent.
(3) The petroleum companies will increase the prices of petrol and diesel by about 5 percent.

Buy Printed Study Material for IBPS, SBI Bank Exams


http://www.bankexamportal.com/study-kit

Page 119

Online Coaching for IBPS, SBI Bank Exams


http://www.bankexamportal.com/elearning
(4) The petrol pumps will stop selling petrol and diesel till the taxes are rolled back by the govt.
(5) None of these
39. Statement : The Govt. has decided to instruct the banks to open new branches in such a way that there is
one branch of any of the banks in every village of population 1000 and above or a cluster of villages with
population less than 1000 to provide banking services to all the citizens. Which of the following will weaken
the step taken by the Govt. ?
(1) The private sector banks in India have stepped up their branch expansion activities in rural India.
(2) Many Govt. owned banks have surplus manpower in its urban branches.
(3) All the banks including those in private sector will follow the govt. directive.
(4) Large number of branches of many Govt. owned banks in the rural areas are making huge losses every
year due to lack of adequate business activities.
(5) None of these
Direction: Study the following information carefully and answer the questions given below.
The centre reportedly wants to continue providing subsidy to consumers for cooking gas and kerosene for
five more years. This is not good news from the point of view of reining in the fiscal deficit. Mounting
subventions for subsidies means diversion of savings by the government from investment to consumption,
raising the cost of capital in the process. The government must cut expenditure on subsidies to create more
fiscal space for investments in both physical and social infrastructure. It should outline a plan for
comprehensive reform in major subsidies including petroleum, food and fertilizers and set goal posts.
40. Which of the following is a conclusion which can be drawn from the facts stated in the above
paragraph ?
(1) Subsidy provided by the government under various heads to the citizen increases the cost of capital.
(2) Govt. is unable to withdraw subsidies provided to various items.
(3) Govt. subsidy on kerosene is purely a political decision.
(4) Govt. does not have enough resources to continue providing subsidy on petroleum products.
(5) None of these
41. Which of the following is an inference which can be made from the facts stated in the above
paragraph ?
(1) Indias fiscal deficit is negligible in Comparison to other emerging economies in the world.
(2) Subsidy on food and fertilizers are essential for growth of Indian economy.
(3) Reform in financial sector will weaken Indias position in the international arena.
(4) Gradual withdrawal of subsidy is essential for effectively managing fiscal deficit in India.
(5) None of these
42. Which of the following is an assumption which is implicit in the facts stated in the above
paragraph ?
(1) People in India may not be able to pay more for petroleum products.
(2) Many people in India are rich enough to buy petroleum products at market cost.
(3) Govt. may not be able to create more infrastructural facilities if the present level of subsidy continues for
a longer time.

Buy Printed Study Material for IBPS, SBI Bank Exams


http://www.bankexamportal.com/study-kit

Page 120

Online Coaching for IBPS, SBI Bank Exams


http://www.bankexamportal.com/elearning
(4) Govt. of India has sought assistance from international financial organizations for its infrastructural
projects
(5) None of these
Direction: Study the following information carefully and answer the questions given below.
Poverty measurement is an unsettled issue, both conceptually and methodologically. Since poverty is a
process as well as an outcome; many come out of it while others may be falling into it. The net effect of
these two parallel processes is a proportion commonly identified as the head count ratio, but these ratios
hide the fundamental dynamism that characterizes poverty in practice. The most recent poverty reestimates
by an expert group has also missed the crucial dynamism. In a study conducted on 13,000 households which
represented the crucial dynamism. In a study conducted on 13,000 households which represented the entire
country in 1993-94 and again on 2004-05, it was found that in the ten-year period 18.2% rural population
moved out of poverty whereas another 22.1 % fell into it over this period. This net increase of about four
percentage points was seen to have a considerable variation across states and regions.
43. Which of the following is a conclusion which can be drawn from the facts stated in the above
paragraph ?
(1) Accurate estimates of number of people living below poverty line in India is possible to be made.
(2) Many expert groups in India are not interested to measure poverty objectively.
(3) Process of poverty measurement needs to take into account various factors to tackle its dynamic nature.
(4) People living below poverty line remain in that position for a very long time.
(5) None of these
44. Which of the following is an assumption which is implicit in the facts stated in the above
paragraph ?
(1) It may not be possible to have an accurate poverty measurement in India.
(2) Level of poverty in India is static over the years.
(3) Researchers avoid making conclusions on poverty measurement data in India.
(4) Govt. of India has a mechanism to measure level of poverty effectively and accurately.
(5) None of these
45. Which of the following is an inference which can be made from the facts stated in the above
paragraph ?
(1) Poverty measurement tools in India are outdated.
(2) Increase in number of persons falling into poverty varies considerably across the country over a period of
time.
(3) Govt. of India has stopped measuring poverty related studies.
(4) People living in rural areas are more susceptible to fall into poverty over the time.
(5) None of these
Direction: In each of the questions given below which one of the five answer figures on the right should
come after the problem figures on the left, if the sequence were continued ?

Buy Printed Study Material for IBPS, SBI Bank Exams


http://www.bankexamportal.com/study-kit

Page 121

Online Coaching for IBPS, SBI Bank Exams


http://www.bankexamportal.com/elearning

Buy Printed Study Material for IBPS, SBI Bank Exams


http://www.bankexamportal.com/study-kit

Page 122

Online Coaching for IBPS, SBI Bank Exams


http://www.bankexamportal.com/elearning

Buy Printed Study Material for IBPS, SBI Bank Exams


http://www.bankexamportal.com/study-kit

Page 123

Online Coaching for IBPS, SBI Bank Exams


http://www.bankexamportal.com/elearning

English Language
Direction : Read the following passage carefully and answer the questions given below it. Certain
words/phrases have been printec in bold to help you locate them while answering some of the questions.
Governments have traditionally equated economic progress with steel mills and cement factories. While urban
centers thrive and city dwellers get rich, hundreds of millions of farmers remain mired in poverty. However,
fears of food shortages, a rethinking of antipoverty priorities and the crushing recession in 2008 are causing a
dramatic shift in world economic policy in favour of greater support for agriculture.
The last time when the worlds farmers felt such love was in the 470s. At that time, as food prices spiked,
there was real concern that the world was facing a crisis in which the planet was simply unable to produce
enough grain and meat for an expanding population. Governments across the developing world and
international aid organisations plowed investment into agriculture in the early 470s, while technological
breakthroughs, like highyield strains of important food crops, boosted production. The result was the Green
Revolution and food production exploded. But the Green Revolution became a victim of its own success. Food
prices plunged by some 60% by the late 480s from their peak in the mid- 470s. Policymakers and aid workers
turned their attention to the poors other pressing needs, such as health care and education. Farming got starved
of resources and investment. By 2004, aid directed at agriculture sank to 3.5% and Agriculture lost its glitter.
Also, as consumers in high-growth giants such as China and India became wealthier, they began eating more
meat, so grain once used for human consumption got diverted to beef up livestock. By early 2008, panicked
buying by importing countries and restrictions slapped on grain exports by some big producers helped drive
prices upto heights not seen for three decades. Making matters worse, land and resources got reallocated to
produce cash crops such as biofuels and the result was that voluminous reserves of grain evaporated. Protests
broke out across the emerging world and fierce food riots toppled governments.
This spurred global leaders into action. This made them aware that food security is one of the fundamental
issues in the world that has to be dealt with in order to maintain administrative and political stability. This also
spurred the U.S. which traditionally provisioned food aid from American grain surpluses to help needy nations,
to move towards investing in farm sectors around the globe to boost productivity. This move helped countries
become more productive for themselves and be in a better position to feed their own people.
Africa, which missed out on the first Green Revolution due to poor policy and limited resources, also
witnessed a change. Swayed by the success of East Asia, the primary povertyfighting method favoured by
many policymakers in Africa was to get farmers off their farms and into modern jobs in factories and urban
centers. But that strategy proved to be highly insufficient. Income levels in the countryside badly trailed those
in cities while the FAO estimated that the number of poor going hungry in 2009 reached an all time high at
more than one billion.
In India on the other hand, with only 40% of its farmland irrigated, entire economic boom currently underway
is held hostage by the unpredictable monsoon. With much of Indias farming areas suffering from drought this
year, the government will have a tough time meeting its economic growth targets. In a report, Goldman Sachs
predicted that if this year too receives weak rains, it could cause agriculture to contract by 2% this fiscal year,
making the governments 7% GDP-growth target look a bit rich. Another green revolution is the need of
the hour and to make it a reality, the global community still has much backbreaking farm work to do.
1. What is the authors main objective in writing the passage

Buy Printed Study Material for IBPS, SBI Bank Exams


http://www.bankexamportal.com/study-kit

Page 124

Online Coaching for IBPS, SBI Bank Exams


http://www.bankexamportal.com/elearning
(1) Criticising developed countries for not bolstering economic growth in poor nations
(2) Analysing the disadvantages of the Green Revolution
(3) Persuading experts that a strong economy depends on industrialization and not agriculture
(4) Making a case for the international society to engineer a second Green Revolution
(5) Rationalising the faulty agriculture policies of emerging countries
2. Which of the following is an adverse impact of the Green Revolution ?
(1) Unchecked crop yields resulted in large tracts of land becoming barren
(2) Withdrawal of fiscal impetus from agriculture to other sectors
(3) Farmers began soliciting government subsidies for their produce
(4) Farmers rioted as food prices fell so low that they could not make ends meet
(5) None of these
3. What is the author trying to convey through the phrase making the governments 7% GDP
growth target look a bit rich ?
(1) India is unlikely to achieve the targeted growth rate
(2) Allocation of funds to agriculture has raised Indias chances of having a high GDP
(3) Agricultural growth has artificially inflated Indias GDP and such growth is not real
(4) India is likely to rave one of the highest GDP growth rates
(5) A large portion of Indias GDP is contributed by agriculture
4. Which of the following factors was/were responsible for the neglect of the farming sector after the
green revolution ?
(A) Steel and cement sectors generated more revenue for the government as compared to agriculture.
(B) Large scale protests against favouring agriculture at the cost of other important sectors such as education
and healthcare.
(C) Attention of policy makers and aid organizations was diverted from agriculture to other sectors.
(1) None (2) Only (C)
(3) Only (B) & (C) (4) Only (A) 8s (B)
(5) All (A), (B) & (C)
5. What prompted leaders throughout the world to take action to boost the agriculture sector in 2008?
(1) Coercive tactics by the U.S. which restricted food aid to poor nations
(2) The realization of the link between food security and political stability
(3) Awareness that performance in agriculture is necessary in order to achieve the targeted GDP
(4) Reports that high-growth countries like China and India were boosting their agriculture sectors to capture
the international markets
(5) Their desire to influence developing nations to slow down their industrial development.
6. What motivated the U.S. to focus on investing in agriculture across the globe ?
(1) To make developing countries become more reliant on U.S. aid
(2) To ensure grain surpluses so that the U.S. had no need to import food
(3) To make those countries more self sufficient to whom it previously provided food
(4) To establish itself in the market before the high-growth giants such as India and China could establish

Buy Printed Study Material for IBPS, SBI Bank Exams


http://www.bankexamportal.com/study-kit

Page 125

Online Coaching for IBPS, SBI Bank Exams


http://www.bankexamportal.com/elearning
themselves
(5) None of these
7. What impact did the economic recession of 2008 have on agriculture ?
(1) Governments equated economic stability with industrial development and shifted away from agriculture
(2) Lack of implementation of several innovative agriculture programmes owing to shortage of funds
(3) It prompted increased investment and interest in agriculture
(4) The GDP as targeted by India was never achieved because of losses in agriculture
(5) None of these
8. What encouraged African policymakers to focus on urban jobs ?
(1) Misapprehension that it would alleviate poverty as it did in other countries
(2) Rural development outstripped urban development in many parts of Africa
(3) Breaking out of protests in the country and the fear that the government would topple
(4) Blind imitation of western models of development
(5) None of these
9. Which of the following had contributed to exorbitant food prices in 2008 ?
(A) Hoarding of food stocks by local wholesalers which inadvertently created a food shortage.
(B) Export of foodgrains was reduced by large producers.
(C) Diverting resources from cultivation of foodgrains to that of more profitable crops.
(1) None (2) Only (C)
(3) Only (B) (4) All (A), (B) & (C)
(5) Only (B) & (C)
10. Which of the following is true about the state of agriculture in India at present ?
(A) Of all the sectors, agriculture needs the highest allocation of funds.
(B) Contribution of agriculture to Indias GDP this year would depend greatly upon the monsoon rains.
(C) As India is one of the high-growth countries, it has surplus food reserves to export to other nations.
(1) Only (A) and (C) (2) Only (C)
(3) Only (B) (4) Only (B) and (C)
(5) None of these
Direction: Choose the word/group of words which is most similar it meaning to the word printed in bold as
used in the passage.
11. STARVED
(1) Deprived (2) Disadvantaged
(3) Hungry (4) Fasting
(5) Emaciated

Buy Printed Study Material for IBPS, SBI Bank Exams


http://www.bankexamportal.com/study-kit

Page 126

Online Coaching for IBPS, SBI Bank Exams


http://www.bankexamportal.com/elearning
12. SLAPPED
(1) Beaten (2) Imposed
(3) Withdrawn (4) Avoided
(5) Persuaded
13. PLOWED
(1) Cultivated (2) Bulldozed
(3) Recovered (4) Instilled
(5) Withdrew
Direction: Choose the word/phrase which is most opposite in meaning to the word printed in bold as used in
the passage.
14. PRESSING
(1) Unpopular (2) Undemanding
(3) Unobtrusive (4) Unsuitable
(5) Unimportant
15. EVAPORATED
(1) Absorbed (2) Accelerated
(3) Grew (4) Plunged
(5) Mismanaged
Direction: Which of the phrases (1), (2),(3) and (4) given below each statement should be placed in the
blank space provided so as to make a meaningful and grammatically correct sentence ? If none of the
sentences is appropriate, mark (5) i.e. None of these as the answer.
16. Refuting the rationale behind frequent agitations for formation of separate States, a recent report
(1) proved that such agitations result in loss of governmental property
(2) indicated that the formation of small states does not necessarily improve the economy
(3) suggested that only large scale agitations have been effective in bringing out desired change in the past
(4) recommended dividing large States into smaller ones to improve governance
(5) None of these
17. Overlooking the fact that water scarcity intensifies during summer,
(1) the government issued guidelines to all builders to limit their consumption to acceptable limits
(2) provision for rainwater harvesting has been made to aid irrigation in drought prone areas
(3) the water table did not improve even after receiving normal monsoon in the current year
(4) many residential areas continue to use swimming pools, wasting large quantities of water
(5) None of these
18. He has lost most of his lifes earning in the stock market but

Buy Printed Study Material for IBPS, SBI Bank Exams


http://www.bankexamportal.com/study-kit

Page 127

Online Coaching for IBPS, SBI Bank Exams


http://www.bankexamportal.com/elearning
(1) He still seems to be leading his life luxuriously and extravagantly
(2) he could not save enough to repay his enormous debts
(3) stock market is not a safe option to invest money unless done with caution
(4) experts have been suggesting to avoid investments in stock market because of its unpredictable nature
(5) None of these
19. Achieving equality for women is not only a laudable goal,
(1) political reforms are also neglected preventing women from entering legislatures and positions of power
(2) the problem is also deep rooted in the society and supported by it
(3) their empowerment is purposefully hampered by people with vested interests in all sections of the society
(4) it is also equally difficult to achieve and maintain for a long term
(5) None of these
20. _______or else they would not keep electing him year after year.
(1) The party leader gave a strong message to the mayor for improving his political style
(2) Owing to numerous scandals against the mayor, he was told to resign from the post immediately
(3) The mayor threatened the residents against filing a complaint against him
(4) The residents must really be impressed with the political style of their mayor
(5) None of these
Direction: Each question below has two blanks, each blank indicating that something has been omitted.
Choose the set of words for each blank that best fits the meaning of the sentence as a whole.
21. Drawing attention to the pitfalls of______solely on Uranium as a fuel for nuclear reactors, Indian
scientists warned that Uranium will not last for long and thus research on Thorium as its____ must be
revived.
(1) using, substitute
(2) believing, replacement
(3) depending, reserve
(4) reckoning, option
(5) relying, alternative
22. In an effort to provide ______ for higher education to all, most of the universities have been
providing education without adequate infrastructure, thus churning out ______ graduates every year.
(1) chances, fresh
(2) platform, capable
(3) opportunities, unemployable
(4) prospects, eligible
(5) policy, incompetent
23. The move to allow dumping of mercury _____ an outcry from residents of the area who _____ that
high levels of mercury will affect their health and destroy ecologically sensitive forest area.

Buy Printed Study Material for IBPS, SBI Bank Exams


http://www.bankexamportal.com/study-kit

Page 128

Online Coaching for IBPS, SBI Bank Exams


http://www.bankexamportal.com/elearning
(1) resulted, insist
(2) provoked, fear
(3) incited, determined
(4) activated, accept
(5) angered, believe
24. _______ has been taken against some wholesale drug dealers for dealing in surgical items without
a valid license and maintaining a stock of _____ drugs.
(1) Note, overwhelming
(2) Step, impressive
(3) Execution, outdated
(4) Action, expired
(5) Lawsuit, invalid
25. Even as the _____ else where in the world are struggling to come out of recession, Indian
consumers are splurging on consumer goods and to _____ this growth, companies are investing
heavily in various sectors.
(1) economies, meet
(2) countries, inhibit
(3) governments, measure
(4) nations, inflict
(5) companies, counter
Direction : Rearrange the following sentences (A), (B), (C), (D), (E) and (F) to make a meaningful paragraph
and then answer the questions which follow
(A) While these disadvantages of bio fuels are serious, they are the only alternate energy source of the future
and the sooner we find solutions to these problems the faster we will be able to solve the problems wo are now
facing with gasoline.
(B) This fuel can also help to stimulate jobs locally since they are also much safer to handle thaw gasoline
and can thus have the potential to turnaround a global economy.
(C) These include dependence on fossil fuels for the machinery required to produce biofuel which ends up
polluting as much as the burning of fossil fuels on roads and exorbitant cost of biofuels which makes it very
difficult for the common man to switch to this option.
(D) This turnaround can potentially help to bring world peace and end the need to depend on foreign countries
for energy requirements.
(E) Biofuels are made from plant sources and since these sources are available in abundance and can be
reproduced on a massive scale they form an energy source that is potentially unlimited.
(F) However everything is not as green with the biofuels as it seems as there are numerous disadvantages
involved which at times overshadow their positive impact.

Buy Printed Study Material for IBPS, SBI Bank Exams


http://www.bankexamportal.com/study-kit

Page 129

Online Coaching for IBPS, SBI Bank Exams


http://www.bankexamportal.com/elearning
26. Which of the following sentence should be the FIFTH after rearrangement ?
(1) A (2) B (3) C
(4) E (5) F
27. Which of the following sentence should be the THIRD after rearrangement ?
(1) A (2) B (3) C
(4) D (5) E
28. Which of the following sentence should be the FIRST after rearrangement ?
(1) A (2) B (3) C
(4) D (5) E
29. Which of the following sentence should be the SIXTH (LAST) after rearrangement ?
(1) A (2) C (3) D
(4) E (5) F
30. Which of the following sentence should be the SECOND after rearrangement ?
(1) A (2) B (3) D
(4) E (5) F
Direction: Which of the phrases (1), (2), (3) and (4) given below each statement should replace the phrase
printed in bold in the sentence to make it grammatically correct ? If the sentence is correct as it is given and
No correction is required, mark (5) as the answer.
31. Soon after the Tsunami had killed thousands of people along the coasts of southern India,
parliament psssas a bill that proposed to set up an institutional mechanism to respond promptly to
natural disasters.
(1) passed a bill that proposed
(2) passes a bill with purpose
(3) pass a bill proposing
(4) passed a bill which propose
(5) No correction required
32. Denial of wages forced scientists and teachers at the agriculture universities throughout the
country to go on strike, crippling crucial research that could help the state of agriculture in the
country.
(1) from going on strike
(2) which went on strike
(3) on going for a strike
(4) for going to strike
(5) No correction required

Buy Printed Study Material for IBPS, SBI Bank Exams


http://www.bankexamportal.com/study-kit

Page 130

Online Coaching for IBPS, SBI Bank Exams


http://www.bankexamportal.com/elearning
33. In an attempt to boost their profits many edible oil producing companies have been engaging
themselves in propaganda against commonly used oils and. promoting exotic and expensive varieties
of oils as more healthier options.
(1) as most healthiest options
(2) as less healthy option
(3) as a healthier option
(4) as much healthiest option
(5) No correction required
34. Thanks to numerous government initiatives, rural masses which was earlier unaware of the
luxuries of urban ways of living are now connected to the same lifestyle.
(1) who was earlier unaware
(2) which were earlier aware
(3) who were earlier conversant
(4) who were earlier unaware
(5) No correction required
35. Over the last few months, while most industries are busy in restructuring operations, cutting costs
and firing, the Indian pharmaceutical and healthcare industry was adding manpower and giving
salary hikes.
(1) as many industries are
(2) while most industries were
(3) while many industries is
(4) where many industries were
(5) No correction required
Direction: In the following passage there are blanks, each of which ]as been numbered. These numbers are
printed below the passage and against each, five words/phrases are suggested, one of which fits the blank
appropriately. Find out the appropriate word/phrase in each case.
There is a considerable amount of research about the factors that make a company innovate. So is it possible
to create an environment (36 ) to innovation? This is a particularly pertinent (37) for India today. Massive
problems in health, education etc. (38) be solved using a conventional approach but (39) creative and
innovative solutions that can ensure radical change and (40). There are several factors in Indias (41). Few
countries have the rich diversity that India or its large, young population (42). While these (43) innovation
policy interventions certain additional steps are also required. These include (44) investment in research and
development by (45) the government and the private sector, easy transfer of technology from the academic
world etc. To fulfill its promise of beng prosperous and to be at the forefront, India must be innovative.
36.
(1) stimuli
(2) conducive
(3) incentive
(4) facilitated
(5) impetus

Buy Printed Study Material for IBPS, SBI Bank Exams


http://www.bankexamportal.com/study-kit

Page 131

Online Coaching for IBPS, SBI Bank Exams


http://www.bankexamportal.com/elearning
37.
(1) objective
(2) controversy
(3) doubt
(4) question
(5) inference
38.
(1) cannot
(2) possibly
(3) should
(4) never
(5) must
39.
(1) necessary
(2) apply
(3) need
(4) consider
(5) requires
40.
(1) quantity
(2) advantages
(3) increase
(4) chaos
(5) growth
41.
(1) challenges
(2) praises
(3) favour
(4) leverage
(5) esteem
42.
(1) blessed
(2) enjoys
(3) endows
(4) prevails
(5) occurs

Buy Printed Study Material for IBPS, SBI Bank Exams


http://www.bankexamportal.com/study-kit

Page 132

Online Coaching for IBPS, SBI Bank Exams


http://www.bankexamportal.com/elearning
43.
(1) aid .
(2) jeopardise
(3) promotes
(4) endure
(5) cater
44.
(1) acute
(2) utilising
(3) restricting
(4) inspiring
(5) increased
45.
(1) both
(2) besides
(3) combining
(4) participating
(5) aid
Direction: In each of the following questions four words are given of which two words are most nearly the
same or opposite in meaning. Find the two words which are most nearly the same or opposite in meaning
and indicate the number of the correct letter combination, by darkening the appropriate oval in your answer
sheet.
46.
(A) consent
(B) nascent
(C) emerging
(D) insecure
(1) AC
(2) BD
(3) BC
(4) AD
(5) AB
47.
(A) elated
(B) eccentric
(C) explicit
(D) abnormal

Buy Printed Study Material for IBPS, SBI Bank Exams


http://www.bankexamportal.com/study-kit

Page 133

Online Coaching for IBPS, SBI Bank Exams


http://www.bankexamportal.com/elearning
(1) AB
(2) BD
(3) AC
(4) AD
(5) DC
48.
(A) abundance
(B) incomparable
(C) projection
(D) plethora
(1) AC
(2) AB
(3) CD
(4) BD
(5) AD
49.
(A) purposefully
(B) inaccurately
(C) inadvertently
(D) unchangeably
(1) AC
(2) AB
(3) BC
(4) BD
(5) AD
50.
(A) germane
(B) generate
(C) reliable
(D) irrelevant
(1) BD
(2) BC
(3) AB
(4) CD
(5) AD

Buy Printed Study Material for IBPS, SBI Bank Exams


http://www.bankexamportal.com/study-kit

Page 134

Online Coaching for IBPS, SBI Bank Exams


http://www.bankexamportal.com/elearning

General Awareness / Marketing / Computers


1. What is the full form of NBFC as used in the Financial Sector ?
(1) New Banking Finance Company
(2) National Banking & Finance Corporation
(3) New Business Finance & Credit
(4) Non Business Fund Company
(5) None of these
2. 100% concession has been given for travelling in the Indian Railways for patients of ...
(1) AIDS (2) Cancer (3) Swine Flu
(4) T. B. (5) None of these
3. Many a times, we read about Special Drawing Right (SDR) in newspapers. As per its definition,
SDR is a monetary unit of the reserve assets of which of the following organizations / agencies?
(1) World Bank
(2) International Monetary Fund (IMF)
(3) Asian Development Bank
(4) Reserve Bank of India
(5) None of these
4. Which of the following is/are the highlights of the Union Budget 20-1 ?
(A) Number of new steps taken to simplify the Foreign Direct Investment (FDI) regime.
(B) Rs. 16,500 crore provided to ensure that the Public Sector Banks are able to attain a minimum 8%
capital (Tier I) by March 201.
(C) More than Rs. 1,74,000 crore provided for the development of the infrastructure in the country.
(1) Only A (2) Only B
(3) Only C (4) All A, B & C
(5) None of these
5. In how many Routes special tourist trains called Bharat Birth is to start?
(1) 19 (2) 16 (3) 17
(4) 18 (5) None of these
6. As per the newspaper reports, the Govt. of India made an auction of the Third Generation
Spectrum (3G) recently. Which of the following ministries was actively involved in the process a
(1) Ministry of Heavy Industries
(2) Ministry of Science & Technology
(3) Ministry of Commerce
(4) Ministry of Foreign Affairs
(5) None of these

Buy Printed Study Material for IBPS, SBI Bank Exams


http://www.bankexamportal.com/study-kit

Page 135

Online Coaching for IBPS, SBI Bank Exams


http://www.bankexamportal.com/elearning
7. As per the recent announcement, the Govt. of India will provide an amount of Rs.48,000 crore to
develop Rural Infrastructure in the country. This planned development is being undertaken under
which of the following schemes ?
(1) Bharat Nirman
(2) Indira Aawas Yojana
(3) Backward Region Grant Fund
(4) Drought Mitigation Fund
(5) None of these
8. Ladies special trains to be renamed with the Name of ...
(1) Bharat Bhoomi Specials
(2) Sonia Gandhi Specials
(3) Matri Bhoomi Specials
(4) Rajiv Gandhi Specials
(5) None of these
9. How much funds has been allocated to the Unique Identification Authority of India?
(1) Rs 1,500 Crore (2) Rs 1,900 Crore
(3) Rs 1,600 Crore (4) Rs 1,800 Crore
(5) None of these
10. What is the rate of Income Tax for incomesz above Rs 1.6 lakh upto Rs 5 lakh?
(1) % (2) 2% (3) 15%
(4) 20% (5) None of these
11. Recently, India took part in Nuclear New Build 20 Conference organized in (1) New Delhi (2) London
(3) Paris (4) Hong Kong
(5) None of these
12. What is the reduction in Service Charges on e-tickets?
(1) Sleeper Class Rs. 20 & AC Class Rs.
(2) Sleeper Class Rs. & AC Class Rs. 20
(3) Sleeper Class Rs. 30 & AC Class Rs. 20
(4) Sleeper Class Rs. 20 & AC Class Rs. 30
(5) None of these
13. As we know, with the launch of Nano by Tatas, India has become favourite Small Car Destination
of the world. Other than India, which of the following countries is also a popular destination of small
cars ?

Buy Printed Study Material for IBPS, SBI Bank Exams


http://www.bankexamportal.com/study-kit

Page 136

Online Coaching for IBPS, SBI Bank Exams


http://www.bankexamportal.com/elearning
(1) Britain (2) France
(3) Germany (4) Thailand
(5) None of these
14. Imports from China in the Year of 2008-09 in Rs...
(1) 0,000 Crore (2) 40,000 Crore
(3) 47,605 Crore (4) 151,000 Crore
(5) None of these
15. How many new teams have been added in IPL 20?
(1) 2 (2) 1 (3) 4
(4) 7 (5) None of these
16. Which countries have recently faced Tsunami Waves?
(1) Japan & Chile (2) Sallie & Korea
(3) China & Thailand (4) Japan & China
(5) None of these
17. North Korea and USA decided to resume their peace talks after a gap of several months. Both the
countries have a dispute over which of the following issues ?
(1) Bailout package offered by USA
(2) Membership of ASEAN to North Korea
(3) Nuclear programme of North Korea
(4) Soaring relations of China with North Korea
(5) None of these
18. Who amongst the following was the Chairperson of the 3th Finance Commission which submitted
its report to the President of India recently ?
(1) Mr. M. V. Kamath (2) Dr. C. Rangarajan
(3) Dr. D. Subbarao (4) Dr. Rakesh Mohan
(5) Dr. Vijay Kelkar
19. As per the news published in various newspapers, the RBI is considering the grant of licence to
some new companies, particularly NBFCs to act as full-fledged banks. Which of the following will be
considered NBFC ?
(1) NABARD
(2) Life Insurance Corporation of India
(3) Reliance Capital
(4) SEBI
(5) None of these
20. Who has scored the highest individual ODI Score?

Buy Printed Study Material for IBPS, SBI Bank Exams


http://www.bankexamportal.com/study-kit

Page 137

Online Coaching for IBPS, SBI Bank Exams


http://www.bankexamportal.com/elearning
(1) Saeed Anwar (Pak)
(2) Charls Coventry (Zim)
(3) Sachin Tendulkar (IND)
(5) Ricky Pointing (Aus)
(5) None of these
21. What is an intranet
(1) Internal internet used to transfer information internally
(2) Internal internet used to transfer information to the outside company
(3) Internal network designed to serve the internal informational needs of a single organization
(4) Internal network designed to transfer the information between two organizations
(5) None of these
22. Which of the following groups of cricket teams was declared joint winner of the Col. C K Naidu
Trophy for 2009 ?
(1) Tamil Nadu & Gujarat
(2) Maharashtra & Kerala
(3) Punjab & Delhi
(4) West Bengal & Maharashtra
(5) None of these
23. Kaiane Aldorino who was crowned Miss World 2009 is from which of the following countries?
(1) Germany (2) Russia
(3) Austria (4) Belgium
(5) Gibraltar
24. Which of the following teams won the Davis Cup Tennis Finals 2009 ?
(1) Germany (2) Spain
(3) Russia (4) France
(5) None of these
25. Who amongst the following is the recipient of the CNN-IBN Indian of the Year Award for 2009?
(1) Ratan Tata (2) A.R.Rahman
(3) Manmohan Singh (4) Sachin Tendulkar
(5) None of these
26. The deficit reduction plan of which of the following countries was reviewed recently in the meeting
of the Finance Ministers of the European Union ?
(1) Germany (2) Romania
(3) Brazil (4) Hungary
(5) Greece

Buy Printed Study Material for IBPS, SBI Bank Exams


http://www.bankexamportal.com/study-kit

Page 138

Online Coaching for IBPS, SBI Bank Exams


http://www.bankexamportal.com/elearning
27. The database administrators function in an organization is
(1) To be responsible for the more technical aspects of managing the information contained in organizational
databases
(2) To be responsible for the executive level aspects of decisions regarding the information management
(3) To show the relationship among entity classes in a data warehouse
(4) To define which data mining tools must be used to extract data
(5) None of these
28. Every device on the Internet has a unique______ address (also called an Internet address) that
identifies it in the same way that a street address identifies the location of a house.
(1) DH (2) DA (3) IP
(4) IA (5) None of these
29. To send another station a message, the main thing a user has to know is
(1) how the network works
(2) the other stations address
(3) whether the network is packet-switched or circuit-switched
(4) whether this is a voice or data network
(5) None of these
30. In a client/ server model, a client program (1) asks for information
(2) provides information and files
(3) serves software files to other computers
(4) distributes data files to other computers
(5) None of these
31. Control in design of an information system is used to
(1) inspect the system and check that it is buiit as per specifications
(2) ensure that the system processes data as it was designed to and that the results are reliable
(3) ensure privacy of data processed by it
(4) protect data from accidental or intentional loss
(5) None of these
32. Each of the following is a true statement except
(1) on-line systems continually update the master file
(2) in on-line processing, the user enters transactions into a device that is directly connected to the computer
system
(3) batch processing is still used today in older systems or in some systems with massive volumes of
transactions
(4) information in batch systems will always be up-to-date
(5) None of these

Buy Printed Study Material for IBPS, SBI Bank Exams


http://www.bankexamportal.com/study-kit

Page 139

Online Coaching for IBPS, SBI Bank Exams


http://www.bankexamportal.com/elearning
33. A set of interrelated components that collect, process, store, and distribute information to support
decision making and control in an organization best defines (1) communications technology
(2) a network
(3) an information system
(4) hardware
(5) None of these
34.A is a computer connected to two networks.
(1) link (2) server
(3) gateway (4) bridge way
(5) None of these
35. When you save a presentation,
(1) all slides in a presentation are saved in the same file
(2) two files are created; one for graphics and one for content
(3) a file is created for each slide
(4) a file is created for each animation or graphic
(5) None of these
36. In a customer database, a customers surname would be keyed into a
(1) row (2) text field
(3) record (4) computed field
(5) None of these
37. Who is the new Prime Minister of Hungry ?
(1) Victor Orban (2) Gorden Bajnai
(3) Jeno Fock (4) Ference Gyurcsany
(5) None of these
38. Storing same data in many places is called
(1) iteration (2) concurrency
(3) redundancy (4) enumeration
(5) None of these
39. Which of the following is the first step in the transaction processing cycle, which captures
business data through various modes such as optical scanning or at an electronic commerce website ?
(1) Document and report generation
(2) Database maintenance
(3) Transaction processing

Buy Printed Study Material for IBPS, SBI Bank Exams


http://www.bankexamportal.com/study-kit

Page 140

Online Coaching for IBPS, SBI Bank Exams


http://www.bankexamportal.com/elearning
(4) Data Entry
(5) None of these
40. CRM (Customer Relationship Management) is
(1) Apre-sales activity
(2) A tool for lead generation
(3) An ongoing daily activity
(4) The task of a DSA
(5) All of the above
41. Who is the new Prime Minister of Denmark ?
(1) Anders Fogh Rasmussen
(2) Lars Looke Rasmussen
(3) Poul Nyrup Rasmussen
(4) Poul Hartling
(5) None of these
42. Who is the Author of the book My China Diary
(1) Kanwal Sibal
(2) Salman Haider
(3) J.N. Dixit
(4) Natwar Singh
(5) None of these
43. One of the following is not involved in the Growth Strategies of a Company (1) Horizontal integration
(2) Vertical integration
(3) Diversification
(4) Intensification (5) None of these
44. A successful Blue Ocean Strategy requires (1) Effective communication
(2) Innovative skills
(3) Motivation
(4) All of the above
(5) None of these
45. Programs from the same developer , sold bundled together , that provide better integration and
share common features , toolbars and menus are known as ....
(1) software suites
(2) integrated software packages
(3) software processing packages

Buy Printed Study Material for IBPS, SBI Bank Exams


http://www.bankexamportal.com/study-kit

Page 141

Online Coaching for IBPS, SBI Bank Exams


http://www.bankexamportal.com/elearning
(4) personal information managers
(5) none of these
46. A data warehouse is which of the following ?
(1) Can be updated by the end users
(2) Contains numerous naming conventions and formats
(3) Organized around important subject areas
(4) Contains only current data
(5) None of these
47. __________ servers store and manages files for network users.
(1) Authentication
(2) Main
(3) Web (4) File
(5) None of these
48. One of the following is not included in the 7 Ps of Marketing. Find the same
(1) Product
(2) Price
(3) Production
(4) Promotion
(5) None of these
49. The target group for SME loans is (1) All Businessmen
(2) All Professionals
(3) All SSIs
(4) All of the above
(5) None of these
50. Home Loans can be best can vassed among (1) Builders
(2) Flat owners
(3) Land developers
(4) Agriculturists
(5) Individuals wanting to buy a flat or house

Buy Printed Study Material for IBPS, SBI Bank Exams


http://www.bankexamportal.com/study-kit

Page 142

Printed Study Material for SBI PO Exam

What you will get:

100% Syllabus Covered

6 Booklets

3000+ MCQs

1100+ Pages

Guidance & Support from Our Experts

Implementation Aspects:

You will Get 6 booklets.

Study Kit will be delivered to your postal Address after payment confirmation.

After dispatching your kit we will provide you a courier tracking details.

For any help we will provide Telephonic & Email Support to the candidates.

http://www.bankexamportal.com/study-kit/sbi-po

Вам также может понравиться